Download as rtf, pdf, or txt
Download as rtf, pdf, or txt
You are on page 1of 52

PN~Comp~Review-CD-1201-1300 1

PN Comprehensive Review CD Questions 1201-1300

{COMP: No Equations/Formulas; <AQ> questions 1251, 1258}

1201. A nurse would look for which one of the following findings in the prenatal client as most
indicative of placental abruption?
1. Severe nausea and vomiting
2. Painless bleeding
3. Tender to rigid abdomen
4. Normal blood pressure
Answer: 3
Rationale: Signs of placental abruption include tender, rigid abdomen; pain, cramp-like to
severe; dark red vaginal bleeding; and maternal shock and fetal distress. The other options are
not findings in placental abruption.
Test-Taking Strategy: To answer this question accurately, you must be familiar with the
manifestations of this clinical problem. Remember, a sign of placental abruption includes a
tender, rigid abdomen. Review the findings in placental abruption if you had difficulty with this
question.
Level of Cognitive Ability: Comprehension
Client Needs: Physiological Integrity
Integrated Process: Nursing Process/Data Collection
Content Area: Maternity/Antepartum
Reference: Leifer, G. (2005). Maternity nursing (9th ed.). Philadelphia: W.B. Saunders, p. 217.

1202. A nurse planning care for a client with placental abruption would plan to:

1. Prepare the client for a cesarean birth


2. Administer frequent enemas until clear
3. Prepare the client for a stress test
4. Reposition the client to the left side
Answer: 1
Rationale: Early diagnosis of placental abruption is critical in managing it effectively. Plans
should be instituted for continuous fetal monitoring, blood work analysis, and either an
immediate cesarean birth or vaginal delivery. The incorrect options are not helpful in managing
this problem.
Test-Taking Strategy: Use the process of elimination. Begin to answer this question by
eliminating options 2 and 3 first, because they are the least likely to be helpful in this situation.
Choose correctly between the remaining options knowing that cesarean birth will deliver the
baby quickly and minimize the risk of maternal and fetal death. Review the interventions for this
complication if you had difficulty with this question.
Level of Cognitive Ability: Application
Client Needs: Physiological Integrity
Integrated Process: Nursing Process/Planning
Content Area: Maternity/Intrapartum
Reference: Leifer, G. (2005). Maternity nursing (9th ed.). Philadelphia: W.B. Saunders, p. 218.
PN~Comp~Review-CD-1201-1300 2

1203. A nurse overhears that a prenatal client is at risk for placental abruption. The nurse would
expect to find which risk factor documented in the client’s record?
1. Gestational diabetes
2. Hyperemesis gravidarum
3. Maternal hypertension
4. Oliguria
Answer: 3
Rationale: It is possible that placental abruption can result from maternal hypertension that
causes degenerative changes in the small arteries that supply intervillous spaces. This results in
thrombosis, causing retroplacental hematoma, and leading to placental separation. Options 1, 2,
and 4 are not specific risk factors for placental abruption.
Test-Taking Strategy: Use the process of elimination and the ABCs—airway, breathing, and
circulation. Option 3 addresses circulatory status. Review the risk factors associated with
placental abruption if you had difficulty with this question.
Level of Cognitive Ability: Comprehension
Client Needs: Physiological Integrity
Integrated Process: Nursing Process/Data Collection
Content Area: Maternity/Antepartum
Reference: Leifer, G. (2005). Maternity nursing (9th ed.). Philadelphia: W.B. Saunders, p. 217.

1204. A nurse is giving instructions to the mother of a preschool child who was recently
diagnosed with pediculosis capitis (head lice). Which item should be included in discussions to
prevent a reinfestation?
1. Machine wash all washable clothing, towels, and bed linens and place in a warm dryer for at
least 20 minutes
2. Shave the child’s hair if pediculocide and nit-removal combs are ineffective
3. Spray the furniture and beds with insecticide
4. Seal nonwashable items in a plastic bag for 2 to 3 weeks in a warm place if they cannot be
vacuumed or dry cleaned
Answer: 4
Rationale: The adult louse can survive up to 48 hours away from a host, while nits can hatch in 7
to 10 days if they are shed into the environment. Thus, 2 weeks represents a safe interval of time
that prevents reinfestation from occurring. Hot water and air are used in the washer and dryer.
Shaving the hair is unnecessary with proper treatment and would have an adverse psychological
effect on the client. Insecticides can endanger children and animals and should not be sprayed on
furniture and beds.
Test-Taking Strategy: Use the process of elimination. Begin to answer this question by
eliminating option 3, because insecticides can endanger children and animals. Eliminate option
2 next because shaving the head would risk psychological harm to the child and is unnecessary.
Knowing that hot water and a hot dryer are recommended, not warm (option 1), will direct you
to option 4. If heat could damage clothing or linen, the items can be sealed in a plastic bag for 2
to 3 weeks in a warm place. Review these teaching points if you had difficulty with this question.
Level of Cognitive Ability: Application
PN~Comp~Review-CD-1201-1300 3

Client Needs: Physiological Integrity

Integrated Process: Teaching/Learning


Content Area: Child Health
Reference: McKinney, E., James, S., Murray, S., & Ashwill, J. (2005). Maternal-child nursing
(2nd ed.). St. Louis: Elsevier, p. 1375.

1205. A generally healthy 63-year-old man is seen in the ambulatory care office for a routine

examination. Which statement by the client would be most important for the nurse to follow-up

on?

1. “Everyone in my immediate family has died from gastrointestinal cancer.”


2. “I try to avoid overly hot or spicy foods because they give me heartburn sometimes.”
3. “I have been following the balanced diet plan that the doctor gave me.”
4. “I check my stool yearly for occult blood.”
Answer: 1
Rationale: The nurse should recognize and follow-up on the statement about familial cancer.

The client may have some anxiety that this will ultimately occur to him, and the nurse should

gather further data to understand the client’s situation. Gathering data about the types of cancer,

age, and sex of affected family members, and the presence of other risk factors provides the

needed information to initiate preventive education. Options 2, 3, and 4 identify appropriate

client statements.

Test-Taking Strategy: Use the process of elimination and note the key words most important for
the nurse to follow-up on. Note that only the correct option is an item suggesting the need for
further data collection. Review data collection techniques and items requiring follow-up care if
you had difficulty with this question.
Level of Cognitive Ability: Comprehension
Client Needs: Physiological Integrity
Integrated Process: Nursing Process/Data Collection
Content Area: Adult Health/Gastrointestinal
Reference: Black, J., & Hawks, J. (2005). Medical-surgical nursing: Clinical management for
positive outcomes (7th ed.). Philadelphia: W.B. Saunders, p. 346.

1206. A client with a tentative diagnosis of gastroesophageal reflux is going to undergo


ambulatory pH monitoring. The nurse should bring which of the following items to the bedside?
1. Intravenous line insertion kit
2. Enema bag
PN~Comp~Review-CD-1201-1300 4

3. Nasogastric (NG) tube


4. Subcutaneous injection syringe
Answer: 3
Rationale: Ambulatory pH monitoring requires insertion of a nasogastric tube. A probe attached
to the nose will assist in monitoring the pH. The other items are unnecessary.
Test-Taking Strategy: Use the process of elimination. Note that the NG tube is the only option
that is directly correlated with the location of the problem. Review this test if you had difficulty
with this question.
Level of Cognitive Ability: Comprehension
Client Needs: Physiological Integrity
Integrated Process: Nursing Process/Implementation
Content Area: Adult Health/Gastrointestinal
References: Black, J., & Hawks, J. (2005). Medical-surgical nursing: Clinical management for
positive outcomes (7th ed.). Philadelphia: W.B. Saunders, pp. 686-687.
Christensen, B., & Kockrow, E. (2003). Foundations of nursing (4th ed.). St. Louis: Mosby, p.
479.

1207. A client has just undergone gastroscopy. Which action would be taken by the nurse as the
most essential postprocedure nursing intervention?
1. Assist the client to gargle with a local anesthetic
2. Keep the client in a prone position
3. Maintain bed rest for the client
4. Check the gag reflex prior to giving oral foods or fluids
Answer: 4
Rationale: To prevent aspiration, the client may not eat or drink after this procedure until
protective airway reflexes return. The nurse must document that the gag and swallow reflexes
have returned. The client would receive a local anesthetic to the throat before the procedure, not
after. Positioning restrictions are not necessary following the procedure.
Test-Taking Strategy: Use the process of elimination and note the key words most essential. Use
the ABCs—airway, breathing, and circulation—to direct you to option 4. Review these
postprocedure interventions if you had difficulty with this question.
Level of Cognitive Ability: Application
Client Needs: Safe, Effective Care Environment
Integrated Process: Nursing Process/Implementation
Content Area: Adult Health/Gastrointestinal
Reference: Chernecky, C., & Berger, B. (2004). Laboratory tests and diagnostic procedures (4th
ed.). Philadelphia: W.B. Saunders, p. 590.

1208. A client who has been taking indomethacin (Indocin) for gout has an order for guaiac
testing of the stool. The nurse explains to the client that this test is necessary because it will
detect which of the following that may be caused or affected by this medication?
1. Steatorrhea
2. Occult blood
3. pH of stool
4. Color of stool
Answer: 2
PN~Comp~Review-CD-1201-1300 5

Rationale: Indomethacin is a nonsteroidal anti-inflammatory medication that can cause


gastrointestinal irritation. The stool guaiac test is noninvasive and is widely used as a gross
screening for blood in the gastrointestinal tract. It is not used for any of the other reasons listed.
Test-Taking Strategy: Specific knowledge of this common test or the side effects of
indomethacin would help you to identify the correct option. Remember, the stool guaiac test is
used as a gross screening for blood in the gastrointestinal tract. Review the purpose of this test if
you had difficulty with this question.
Level of Cognitive Ability: Application
Client Needs: Physiological Integrity
Integrated Process: Nursing Process/Implementation
Content Area: Adult Health/Gastrointestinal
Reference: Pagana, K., & Pagana, T. (2003). Mosby’s diagnostic and laboratory test reference
(6th ed.). St. Louis: Mosby, p. 830.

1209. A client who has undergone a barium enema is being readied for discharge. The nurse
determines that the client has understood discharge instructions if the client states:
1. “I will call if I haven’t had a normal bowel movement in a week.”
2. “I need to take a laxative every night from now on.”
3. “I will continue on a low residue diet for several days and limit fluids.”
4. “I should take a laxative and my stool should return to normal color.”
Answer: 4
Rationale: Discharge teaching following this procedure includes that the client should take a
laxative to enhance passage of remaining barium from the bowel to prevent impaction. Stools
change from clay-colored back to a normal color once all barium is eliminated. The information
contained in the client’s other statements does not reflect accurate discharge teaching.
Test-Taking Strategy: Focus on the issue of the question, which is knowledge of items that
should be reviewed as part of discharge teaching following barium enema. Recalling the effects
of barium will direct you to option 4. Review these postprocedure instructions if you had
difficulty with this question.
Level of Cognitive Ability: Comprehension
Client Needs: Health Promotion and Maintenance
Integrated Process: Nursing Process/Evaluation
Content Area: Adult Health/Gastrointestinal
Reference: Chernecky, C., & Berger, B. (2004). Laboratory tests and diagnostic procedures (4th
ed.). Philadelphia: W.B. Saunders, p. 219.

1210. A client with a possible hiatal hernia complains of frequent heartburn and regurgitation.
The nurse should gather further information about the presence of which sign or symptom of
hiatal hernia?
1. Dizziness after meals
2. Moderate right upper quadrant pain unrelated to eating
3. Difficulty swallowing
4. Left lower quadrant pain 2 hours after eating
Answer: 3
PN~Comp~Review-CD-1201-1300 6

Rationale: Although many clients with hiatal hernia are asymptomatic, those with symptoms
usually have difficulty swallowing along with heartburn and reflux. Options 1, 2, and 4 are not
related to this disorder.
Test-Taking Strategy: To answer this question correctly, recall that this client has an upper
gastrointestinal disorder. Knowing that pain is usually in the epigastric area, and that it
correlates with food intake, will assist in eliminating each of the incorrect options. Review the
manifestations associated with this disorder if you had difficulty with this question.
Level of Cognitive Ability: Comprehension
Client Needs: Physiological Integrity
Integrated Process: Nursing Process/Data Collection
Content Area: Adult Health/Gastrointestinal
Reference: Linton, A., & Maebius, N. (2003). Introduction to medical-surgical nursing (3rd
ed.). Philadelphia: W.B. Saunders, p. 680.

1211. A nurse is teaching a client with a newly diagnosed hiatal hernia about measures to
prevent reoccurrence of symptoms. Which statement would be included in the teaching?
1. “Be sure to sleep with your head elevated in bed.”
2. “This problem requires surgery.”
3. “Eat foods that are higher in fat to slow down digestion.”
4. “Lie down for at least an hour after eating.”
Answer: 1
Rationale: Most clients with hiatal hernia can be managed by conservative measures, which
include a low-fat diet, avoiding lying down for an hour after eating, and raising the head of the
bed.
Test-Taking Strategy: Use the process of elimination. Note that options 1 and 4 seem to oppose
each other, making it more likely that one of them is correct. Use knowledge of this health
problem to direct you to option 1. Review this disorder if you had difficulty with this question.
Level of Cognitive Ability: Application
Client Needs: Physiological Integrity
Integrated Process: Teaching/Learning
Content Area: Adult Health/Gastrointestinal
Reference: Linton, A., & Maebius, N. (2003). Introduction to medical-surgical nursing (3rd
ed.). Philadelphia: W.B. Saunders, p. 680.

1212. A client will undergo a barium swallow to determine whether the client has a hiatal hernia.
The nurse instructs the client to:
1. Avoid eating or drinking after midnight before the test
2. Have a clear liquid breakfast on the morning of the test
3. Take all routine medications on the morning of the test
4. Limit self to two cigarettes on the morning of the test
Answer: 1
Rationale: The stomach should be empty at the time of a barium swallow, because food and
medications can interfere with test results. Smoking increases mucus and acid production and
can also interfere with the test. For this reason, all foods, liquids, medication, and smoking are
avoided before the test.
PN~Comp~Review-CD-1201-1300 7

Test-Taking Strategy: Use the process of elimination and focus on the issue—preparation for a
barium swallow. Remember that options that are similar are not likely to be correct. Each of the
incorrect options involves taking in something on the morning of the examination. Review
preparation for this test if you had difficulty with this question.
Level of Cognitive Ability: Application
Client Needs: Physiological Integrity
Integrated Process: Nursing Process/Implementation
Content Area: Adult Health/Gastrointestinal
Reference: Chernecky, C., & Berger, B. (2004). Laboratory tests and diagnostic procedures (4th
ed.). Philadelphia: W.B. Saunders, p. 220.

1213. A clinic nurse documents that a client with a hiatal hernia is implementing effective health
maintenance measures after the client reports during a subsequent visit doing which of the
following?
1. Eating low-fat or non-fat foods
2. Elevating the foot of the bed during sleep
3. Doing household chores immediately after eating
4. Sleeping with the head of the bed slightly down
Answer: 1
Rationale: The consumption of low-fat or non-fat foods is recommended to reduce gastric
pressure and prevent sliding of the hernia through the cardiac sphincter. The client should also
elevate the head of the bed during sleep, and wait at least 1 hour after meals to perform chores.
Test-Taking Strategy: Use the process of elimination. Remember that options that are similar
are not likely to be correct. With this in mind, eliminate options 2 and 4. Choose correctly
between the remaining options knowing that low-fat foods are helpful, while activity following
meals can aggravate reflux. Review these measures if you had difficulty with this question.
Level of Cognitive Ability: Analysis
Client Needs: Health Promotion and Maintenance
Integrated Process: Nursing Process/Evaluation
Content Area: Adult Health/Gastrointestinal
Reference: Linton, A., & Maebius, N. (2003). Introduction to medical-surgical nursing (3rd
ed.). Philadelphia: W.B. Saunders, p. 682.

1214. A client arrives in the emergency department with bleeding esophageal varices, and the
physician states that a Sengstaken-Blakemore tube will be used to try to control gastrointestinal
hemorrhage. The nurse prepares for insertion via which of the following routes?
1. Percutaneous
2. Orogastric
3. Nasogastric
4. Gastrostomy
Answer: 3
Rationale: A Sengstaken-Blakemore tube is inserted via the nose into the esophagus and
stomach. The other options are incorrect.
Test-Taking Strategy: To answer this question accurately, you must be familiar with this type of
tube and its use. Eliminate options 1 and 4 first because they are least likely to be correct.
PN~Comp~Review-CD-1201-1300 8

Choose between the remaining options after visualizing the tube and what it is intended to
accomplish. Review this type of tube if you had difficulty with this question.
Level of Cognitive Ability: Comprehension
Client Needs: Physiological Integrity
Integrated Process: Nursing Process/Planning
Content Area: Adult Health/Gastrointestinal
Reference: Linton, A., & Maebius, N. (2003). Introduction to medical-surgical nursing (3rd
ed.). Philadelphia: W.B. Saunders, p. 661.

1215. A nurse is caring for a client with a Sengstaken-Blakemore tube. To prevent ulceration
and necrosis of oral and nasal mucosa, the nurse should plan to:
1. Have a family member remain with the client
2. Keep scissors at the bedside
3. Give frequent oral and nasal care
4. Provide tracheal suction as needed
Answer: 3
Rationale: The issue of the question is knowledge that frequent oral care, including oral
suctioning, is necessary to prevent irritation to the oral and nasal mucosa. A family member’s
presence will not prevent this from occurring, nor will the actions taken in options 2 and 4.
Keeping scissors at the bedside is a good action; however, these are used to cut the tube if the
client begins to have airway maintenance problems.
Test-Taking Strategy: Focus on the issue of the question, which is prevention of necrosis and
ulceration of oral and nasal mucosa. Eliminate each of the incorrect options because they do not
address this problem. Note also the presence of the words “oral” and “nasal” in both the
question and the correct option. Review care to the client with a Sengstaken-Blakemore tube if
you had difficulty with this question.
Level of Cognitive Ability: Application
Client Needs: Physiological Integrity
Integrated Process: Nursing Process/Planning
Content Area: Adult Health/Gastrointestinal
Reference: Linton, A., & Maebius, N. (2003). Introduction to medical-surgical nursing (3rd
ed.). Philadelphia: W.B. Saunders, p. 661.

1216. A client with a Sengstaken-Blakemore tube in place to treat esophageal varices suddenly
becomes restless, the client’s heart rate and blood pressure increase, and the client is having
difficulty breathing. The immediate nursing action would be to:
1. Cut the tube and pull it out
2. Administer oxygen at 3 L per minute via nasal cannula
3. Call respiratory therapy
4. Stay with the client and use the call bell to summon help
Answer: 1
Rationale: Sudden rupture of the esophageal balloon can cause airway obstruction, aspiration,
and/or asphyxiation. The tube should be cut and removed to prevent airway obstruction.
Options 3 and 4 may also be done, once the nurse first takes action to safeguard the client’s
airway. Option 2 may be completed with a physician’s order, again at a slightly later time.
PN~Comp~Review-CD-1201-1300 9

Test-Taking Strategy: Use the process of elimination and note the key word immediate. This
tells you that more than one or all actions may be partially or totally correct, but that one is better
than the others. Use the ABCs—airway, breathing, and circulation. Only option 1 definitively
assists in maintaining the client’s airway. Review care to the client with a Sengstaken-
Blakemore tube if you had difficulty with this question.
Level of Cognitive Ability: Application
Client Needs: Physiological Integrity
Integrated Process: Nursing Process/Implementation
Content Area: Adult Health/Gastrointestinal
Reference: Black, J., & Hawks, J. (2005). Medical-surgical nursing: Clinical management for
positive outcomes (7th ed.). Philadelphia: W.B. Saunders, pp. 1345-1346.

1217. A 70-year old client complains of stomach pain 30 minutes to 1 hour after eating. The
pain is not relieved by further intake of food, although it is relieved by vomiting. A gastric ulcer
is suspected. Which of the following data would further support this diagnosis?
1. Complaints of stress with a history of chronic renal failure
2. Frequent heartburn with a sour taste in the mouth
3. History of chronic obstructive pulmonary disease with weight gain
4. History of alcohol use, smoking, and weight loss
Answer: 4
Rationale: Alcohol use, smoking, and weight loss are most commonly associated with gastric
ulcers. The other options do not contain risk factors or symptoms commonly associated with this
disorder. The symptoms listed in option 2 may be seen in gastroesophageal reflux disease.
Test-Taking Strategy: Use the process of elimination and focus on the issue—data that would
support this diagnosis. Recalling that alcohol use and smoking can lead to gastric ulcer
formation and that weight loss is part of the clinical picture will direct you to option 4. Review
the manifestations associated with a gastric ulcer if you had difficulty with this question.
Level of Cognitive Ability: Comprehension
Client Needs: Physiological Integrity
Integrated Process: Nursing Process/Data Collection
Content Area: Adult Health/Gastrointestinal
Reference: Christensen, B., & Kockrow, E. (2003). Adult health nursing (4th ed.). St. Louis:
Mosby, p. 184.

1218. A client with peptic ulcer disease has been prescribed to take misoprostol (Cytotec) and
sucralfate (Carafate). The nurse teaches the client that these two medications will work primarily
to:
1. Inhibit histamine action
2. Kill bacteria
3. Decrease stomach acid
4. Protect the gastric mucosa
Answer: 4
Rationale: Both of these medications protect the stomach lining. Misoprostol increases mucus
production and bicarbonate levels, while sucralfate coats the ulcer surface. Options 1 and 2
reflect histamine antagonists and antibiotics, respectively. Option 3 describes antacids.
PN~Comp~Review-CD-1201-1300 10

Test-Taking Strategy: Specific knowledge of the mechanism of action for these medications is
needed to answer this question. Remember that both of these medications protect the stomach
lining. Review these medications if you had difficulty with this question.
Level of Cognitive Ability: Application
Client Needs: Health Promotion and Maintenance
Integrated Process: Teaching/Learning
Content Area: Adult Health/Gastrointestinal
Reference: Hodgson, B., & Kizior, R. (2005). Saunders nursing drug handbook 2005.
Philadelphia: W.B. Saunders, pp. 721, 994.

1219. A nurse would include which of the following when reinforcing home care instructions for
a client whose has peptic ulcer disease?
1. Continue to eat the same diet as before the diagnosis
2. Smoke only at bedtime
3. Learn to use stress reduction techniques
4. Take nonsteroidal anti-inflammatory drugs (NSAIDs) for pain relief
Answer: 3
Rationale: Identifying and reducing stress is essential to a comprehensive ulcer management
plan. The client should also limit intake of foods that aggravate pain, quit smoking, and avoid
irritants such as NSAIDs. Antibiotic therapy often cures the client of this problem in many
instances.
Test-Taking Strategy: Use the process of elimination. The issue of the question is knowledge of
factors that will reduce symptoms of peptic ulcer disease. Eliminate each of the incorrect options
knowing that only option 3 is consistent with minimizing the disease process. Review these
treatment measures if you had difficulty with this question.
Level of Cognitive Ability: Application
Client Needs: Health Promotion and Maintenance
Integrated Process: Teaching/Learning
Content Area: Adult Health/Gastrointestinal
Reference: Christensen, B., & Kockrow, E. (2003). Adult health nursing (4th ed.). St. Louis:
Mosby, p. 193.

1220. A nurse is reinforcing dietary instructions to a client with peptic ulcer disease. The nurse
tells the client to:
1. Include foods that will increase gastrointestinal (GI) motility
2. Eat six small meals every day
3. Consume a bland diet only
4. Eat anything as long as it does not aggravate or cause pain
Answer: 4
Rationale: The client may eat any food as long as it does not aggravate or cause pain. Increased
GI motility should be avoided. A traditional bland diet is no longer recommended. It is
unnecessary for the client to eat six small meals per day with this disorder, although smaller
sized meals are better managed by the client.
Test-Taking Strategy: Use the process of elimination. Option 1 can be eliminated first knowing
that GI motility should not be increased. Next eliminate options 2 and 3 because of the absolute
PN~Comp~Review-CD-1201-1300 11

words “every” and “only.” Review dietary measures for this disorder if you had difficulty with
this question.
Level of Cognitive Ability: Application
Client Needs: Health Promotion and Maintenance
Integrated Process: Teaching/Learning
Content Area: Adult Health/Gastrointestinal
References: Christensen, B., & Kockrow, E. (2003). Adult health nursing (4th ed.). St. Louis:
Mosby, p. 187.
Linton, A., & Maebius, N. (2003). Introduction to medical-surgical nursing (3rd ed.).
Philadelphia: W.B. Saunders, p. 686.

1221. A nurse who has assisted in dietary teaching for a client with peptic ulcer disease sees the
client at a routine follow-up visit. The nurse documents which of the following as the best
indicator of a successful outcome for this client?
1. Need for continued PRN medications
2. A decrease in sour eructation
3. Use of only decaffeinated coffee and tea
4. Taking in increased dairy products
Answer: 2
Rationale: A decrease in sour eructation (burping) represents a change in the client’s health
status and is an effective indicator of a successful outcome. Option 3 represents healthy behavior
by the client but is not focused on outcome specific to this disease. Options 1 and 4 are not
consistent with minimizing disease symptoms.
Test-Taking Strategy: Use the process of elimination and focus on the issue—a successful
outcome. Begin to answer this question by eliminating options 1 and 4, which are not
compatible with improved health status. Choose option 2 over 3 by focusing on the
manifestations of this disease and the key word outcome. Review this disease if you had
difficulty with this question.
Level of Cognitive Ability: Analysis
Client Needs: Physiological Integrity
Integrated Process: Nursing Process/Evaluation
Content Area: Adult Health/Gastrointestinal
References: Christensen, B., & Kockrow, E. (2003). Adult health nursing (4th ed.). St. Louis:
Mosby, p. 187.
Linton, A., & Maebius, N. (2003). Introduction to medical-surgical nursing (3rd ed.).
Philadelphia: W.B. Saunders, p. 686.

1222. A preschool child who was admitted to the hospital with a serious respiratory tract
infection develops a rash on the second day after hospitalization and is diagnosed with
chickenpox (varicella). The nurse should take which of the following actions to provide safety
for all children on the unit?
1. Place the child and other children who were exposed to the infected child in isolation
2. Keep siblings from visiting the infected child
3. Place only the infected child in isolation
4. Place the child in isolation and isolate immunocompromised children from the infected child
Answer: 4
PN~Comp~Review-CD-1201-1300 12

Rationale: The period of communicability for chickenpox is 1 day before the eruption of
vesicles to about 1 week when crusts are formed. The infected child should be isolated until
vesicles have dried, and other high-risk children (immunocompromised) should be isolated from
the infected child.
Test-Taking Strategy: Use the process of elimination. Recalling the incubation period and the
period of communicability for chickenpox will direct you to option 4. Review this
communicable disease if you had difficulty with this question.
Level of Cognitive Ability: Application
Client Needs: Safe, Effective Care Environment
Integrated Process: Nursing Process/Implementation
Content Area: Child Health
Reference: Leifer, G. (2003). Introduction to maternity & pediatric nursing (4th ed.).
Philadelphia: W.B. Saunders, p. 749.

1223. A client has undergone a subtotal gastrectomy, and the nurse is preparing the client for
discharge. Which item should the nurse, who is assisting in teaching the client about ongoing
self-management, tell the client?
1. Stress can no longer exacerbate gastrointestinal symptoms
2. The client can resume full activity immediately
3. Follow-up visits with the physician are no longer needed
4. Smaller, more frequent meals should be eaten
Answer: 4
Rationale: Following gastric surgery, the client should eat smaller, more frequent meals to
facilitate digestion. The client should resume activity gradually, and should minimize stressors
to prevent recurrence of symptoms. The client requires ongoing medical supervision and
evaluation.
Test-Taking Strategy: Use the process of elimination. Option 3 can be eliminated first knowing
that follow-up care is important. From the remaining options, recall that subtotal gastrectomy is
often used to manage more severe gastric ulcers. With this in mind, eliminate the options that
would actually increase gastrointestinal symptoms. Review care to the client following subtotal
gastrectomy if you had difficulty with this question.
Level of Cognitive Ability: Application
Client Needs: Health Promotion and Maintenance
Integrated Process: Teaching/Learning
Content Area: Adult Health/Gastrointestinal
Reference: Linton, A., & Maebius, N. (2003). Introduction to medical-surgical nursing (3rd
ed.). Philadelphia: W.B. Saunders, p. 687.

1224. A nurse who is reinforcing instructions to a client following gastric resection should be
sure to include which of the following suggestions?
1. Eat a diet high in vitamin B12 content
2. Avoid iron supplementation
3. Self-monitor for signs and symptoms of lower gastrointestinal (GI) hemorrhage
4. Take action to prevent dumping syndrome
Answer: 4
PN~Comp~Review-CD-1201-1300 13

Rationale: Dumping syndrome occurs in many clients after GI surgery and may occur as an
early or late complication. Upper GI hemorrhage may also occur. A diet high in vitamin B12 will
not prevent pernicious anemia, because the client lacks intrinsic factor needed for absorption.
Instead, the client requires injection to supplement this vitamin. Iron supplements are necessary
to aid the absorption of parenteral vitamin B12.
Test-Taking Strategy: Use the process of elimination. Recalling the complications that occur
following gastric resection will direct you to option 4. Review client instructions following this
procedure if you had difficulty with this question.
Level of Cognitive Ability: Application
Client Needs: Health Promotion and Maintenance
Integrated Process: Teaching/Learning
Content Area: Adult Health/Gastrointestinal
Reference: Christensen, B., & Kockrow, E. (2003). Adult health nursing (4th ed.). St. Louis:
Mosby, p. 190.

1225. A nurse reinforces teaching with a client following gastrectomy about the signs and
symptoms of pernicious anemia knowing that:
1. Once symptoms are evident, pernicious anemia is fatal
2. Regular monthly injections of vitamin B12 will prevent this complication
3. Symptoms can occur as long as 10 years after surgery
4. Most diets are deficient in all of the B vitamins
Answer: 2
Rationale: Vitamin B12 deficiency occurs from lack of intrinsic factor normally secreted by
specialized cells in the gastric mucosa. Replacement therapy is given by the parenteral route.
Symptoms generally occur within 5 years or less. Although not fatal, pernicious anemia can
contribute to many other diseases.
Test-Taking Strategy: Focus on the issue—pernicious anemia. Recalling that this disorder is
treated with monthly injections of vitamin B12 will direct you to option 2. Review this disorder if
you had difficulty with this question.
Level of Cognitive Ability: Comprehension
Client Needs: Physiological Integrity
Integrated Process: Teaching/Learning
Content Area: Adult Health/Gastrointestinal
Reference: Christensen, B., & Kockrow, E. (2003). Adult health nursing (4th ed.). St. Louis:
Mosby, p. 190.

1226. A nurse is assisting in the care of a client within the first 24 hours following a total
gastrectomy for gastric cancer. The nurse should focus interventions on which of the following
during this timeframe?
1. Maintaining a patent nasogastric (NG) tube
2. Providing the client with an oral diet
3. Teaching symptoms of dumping syndrome
4. Promoting the use of stress reduction techniques
Answer: 1
Rationale: An NG tube is inserted during surgery and is left in place for 24 to 48 hours to
decompress the gastrointestinal tract, which enhances sealing of the suture line. It is essential
PN~Comp~Review-CD-1201-1300 14

that the NG tube does not become occluded because this could disrupt the suture lines if
distention occurs. The other options are also appropriate, but at a later time.
Test-Taking Strategy: Use the process of elimination. The key words are during this timeframe.
This tells you that more than one or all of the options may be partially or totally correct, but that
one of them is best given the time period provided. Focusing on these key words will direct you
to option 1. Review care to the client following gastric surgery if you had difficulty with this
question.
Level of Cognitive Ability: Application
Client Needs: Physiological Integrity
Integrated Process: Nursing Process/Implementation
Content Area: Adult Health/Gastrointestinal
Reference: Christensen, B., & Kockrow, E. (2003). Adult health nursing (4th ed.). St. Louis:
Mosby, pp. 190-191.

1227. A nurse is collecting data about how well a client with a gastrointestinal (GI) disorder is
able to absorb food. While carrying out this function, the nurse recalls that absorption is best
defined as:
1. The transfer of nutrients into the cell by active transport
2. A chemical process involving the breakdown of foods
3. Removal by osmosis of digested food to the cells
4. The transfer of digested food molecules from the GI tract into the bloodstream
Answer: 4
Rationale: Absorption is the transfer of digested food molecules into the bloodstream. The
blood then carries nutrients to the cells. Active transport is the process used to transfer nutrients
into the cells. Digestion involves the mechanical and chemical breakdown of foods. Option 3 is
an incorrect statement.
Test-Taking Strategy: Focus on the issue—absorption. Recalling the definition of this word will
direct you to option 4. Review this definition if you had difficulty with this question.
Level of Cognitive Ability: Knowledge
Client Needs: Physiological Integrity
Integrated Process: Nursing Process/Data Collection
Content Area: Adult Health/Gastrointestinal
Reference: Linton, A., & Maebius, N. (2003). Introduction to medical-surgical nursing (3rd
ed.). Philadelphia: W.B. Saunders, pp. 73-74.

1228. The nurse is preparing to collect data about a client by examining the abdomen. The
nurse should assist the client into which of the following positions first?
1. Sims position
2. Supine with the head and feet flat
3. Supine with the head raised slightly and the knees slightly flexed
4. Semi-Fowler’s with the head raised 45 degrees and the knees flat
Answer: 3
Rationale: In the supine position, the client lies on the back, and the abdominal muscles are
relaxed by elevating the head and flexing the knees slightly. If the head was raised to 45 degrees,
the abdomen could not be accurately assessed. Sims position is a side-lying position and would
PN~Comp~Review-CD-1201-1300 15

not adequately expose the abdomen for examination. Placing the head and feet flat would result
in the abdominal muscles being taut.
Test-Taking Strategy: Use the process of elimination. Visualize each of the positions identified
and evaluate each of them against their natural ability to keep abdominal muscles relaxed.
Review this data collection technique if you had difficulty with this question.
Level of Cognitive Ability: Application
Client Needs: Health Promotion and Maintenance
Integrated Process: Nursing Process/Data Collection
Content Area: Adult Health/Gastrointestinal
Reference: Black, J., & Hawks, J. (2005). Medical-surgical nursing: Clinical management for
positive outcomes (7th ed.). Philadelphia: W.B. Saunders, pp. 779-780.

1229. A licensed practical nurse (LPN) is reinforcing follow-up teaching after a client underwent
an upper gastrointestinal (GI) series. The nurse reminds the client that the stools will remain
white for approximately:
1. One day
2. Seven days
3. One week
4. Ten days
Answer: 1
Rationale: It takes at least 12 to 24 hours for a substance to pass through the colon. The other
timeframes listed are excessive in length. Barium should be eliminated quickly to reduce the risk
of impaction from this substance.
Test-Taking Strategy: Use the process of elimination. Eliminate options 2 and 3 first because
they are similar. From the remaining options, eliminate option 4 because the timeframe is
excessive in length. Review this diagnostic test if you had difficulty with this question.
Level of Cognitive Ability: Application
Client Needs: Health Promotion and Maintenance
Integrated Process: Nursing Process/Implementation
Content Area: Adult Health/Gastrointestinal
Reference: Chernecky, C., & Berger, B. (2004). Laboratory tests and diagnostic procedures (4th
ed.). Philadelphia: W.B. Saunders, p. 1109.

1230. A physician asks the nurse to obtain a Salem sump tube for gastric intubation. The nurse
would correctly select which of the following tubes from the unit storage area?
1. A Sengstaken-Blakemore tube
2. A nasogastric tube
3. A tube with just a single lumen
4. A tube with a larger lumen and an air vent
Answer: 4
Rationale: A Salem sump is used commonly for gastric intubation, and has a larger suction
lumen as well as a smaller air vent. Options 1 describes a tube used for gastroesophageal
bleeding. Options 2 and 3 describe a Levin tube.
Test-Taking Strategy: Use the process of elimination. Eliminate options 2 and 3 because they
are similar. From the remaining options, recalling that a Sengstaken-Blakemore tube has a
PN~Comp~Review-CD-1201-1300 16

balloon that controls bleeding will direct you to option 4. Review these types of tubes if you had
difficulty with this question.
Level of Cognitive Ability: Application
Client Needs: Physiological Integrity
Integrated Process: Nursing Process/Implementation
Content Area: Fundamental Skills
Reference: Linton, A., & Maebius, N. (2003). Introduction to medical-surgical nursing (3rd
ed.). Philadelphia: W.B. Saunders, 661.

1231. A nurse has assisted to insert a Levin tube for gastrointestinal (GI) decompression. The
nurse plans to set the suction to which of the following pressures?
1. Low and continuous
2. High and intermittent
3. Low and intermittent
4. High and continuous
Answer: 3
Rationale: A Levin tube has no air vent, and the suction must be placed on an intermittent setting
to prevent trauma to the gastric mucosa. Low pressure and intermittent suction are safer for the
stomach than high pressure and continuous suction.
Test-Taking Strategy: Use the process of elimination. Begin to answer this question by
eliminating options 2 and 4 because of the word “high.” To choose between the remaining
options, you must recall that a Levin tube has no air vent. This will direct you to option 3.
Review the procedure for GI decompression if you had difficulty with this question.
Level of Cognitive Ability: Application
Client Needs: Physiological Integrity
Integrated Process: Nursing Process/Planning
Content Area: Adult Health/Gastrointestinal
References: Black, J., & Hawks, J. (2005). Medical-surgical nursing: Clinical management for
positive outcomes (7th ed.). Philadelphia: W.B. Saunders, p. 745.
Linton, A., & Maebius, N. (2003). Introduction to medical-surgical nursing (3rd ed.).
Philadelphia: W.B. Saunders, p. 661.

1232. A nurse is caring for a client with anorexia. The nurse plans care for the client focusing
on which of the following as the primary problem?
1. Lack of nutritional knowledge
2. Impaired nutritional status
3. Depression
4. Pain
Answer: 2
Rationale: A client with anorexia has decreased appetite, and this could be due to any number of
causes. The nurse plans care focusing on the risk of impaired nutritional status. The other
options listed may or may not contribute to the client’s problem.
Test-Taking Strategy: Use Maslow’s Hierarchy of Needs theory to answer the question. This
will direct you to option 2. Review the complications of anorexia if you had difficulty with this
question.
Level of Cognitive Ability: Application
PN~Comp~Review-CD-1201-1300 17

Client Needs: Physiological Integrity


Integrated Process: Nursing Process/Planning
Content Area: Adult Health/Gastrointestinal
Reference: Linton, A., & Maebius, N. (2003). Introduction to medical-surgical nursing (3rd
ed.). Philadelphia: W.B. Saunders, p. 670.

1233. A client has been diagnosed with chronic gastritis and has been told that there is too little
intrinsic factor being produced. The nurse who is reinforcing client teaching tells the client
about the need for:
1. Vitamin B12 injections
2. Vitamin B6 injections
3. Antibiotic therapy
4. Antacid use
Answer: 1
Rationale: Insufficient intrinsic factor results in inability to absorb vitamin B12, which must then
be supplemented by the parenteral route. Vitamin B6 is absorbed when given orally. Antibiotic
therapy and antacid use would not help to treat lack of intrinsic factor.
Test-Taking Strategy: Note the key words little intrinsic factor being produced. Use knowledge
of anatomy and physiology to direct you to option 1. Review this content if you had difficulty
with this question.
Level of Cognitive Ability: Comprehension
Client Needs: Physiological Integrity
Integrated Process: Nursing Process/Implementation
Content Area: Adult Health/Gastrointestinal
Reference: Christensen, B., & Kockrow, E. (2003). Adult health nursing (4th ed.). St. Louis:
Mosby, p. 190.

1234. A nurse is assisting in the care of a client in the emergency department who has right
lower quadrant pain. After noting a white blood cell count of 16,500/mm3, the nurse would
question an order for which of the following?
1. NPO
2. Intravenous (IV) fluids at a rate of 100 mL per hour
3. Cold pack to the abdomen
4. Milk of Magnesia
Answer: 4
Rationale: A client with right lower quadrant pain may have appendicitis. This client would be
NPO and given IV fluids for hydration. Cold packs may provide comfort. Laxatives are not
ordered; therefore the nurse would question the Milk of Magnesia order.
Test-Taking Strategy: Use the process of elimination. The key words in the question are
question an order for. This tells you that correct option is an incorrect order for treatment of this
client. Focusing on the data in the question will assist in directing you to option 4. Review care
for the client with appendicitis if you had difficulty with this question.
Level of Cognitive Ability: Analysis
Client Needs: Physiological Integrity
Integrated Process: Nursing Process/Implementation
Content Area: Adult Health/Gastrointestinal
PN~Comp~Review-CD-1201-1300 18

Reference: Christensen, B., & Kockrow, E. (2003). Adult health nursing (4th ed.). St. Louis:
Mosby, p. 205.

1235. A nurse receives a call from the mother of a child who has a foreign body in the eye,
which is clearly visible and is not imbedded. The mother asks the nurse what is the most
effective way to get it out. The nurse would respond that which of the following methods would
be most useful?
1. Let the object just “work its way out”
2. Touch the object gently with a cotton swab and lift it out
3. Irrigate the eye with natural tears
4. Irrigate the eye with running tap water
Answer: 2
Rationale: The most effective method that would cause the least amount of trauma would be to
lift the foreign body from the eye. It should not be allowed to remain and “work its way out.”
Irrigating the eye may cause the foreign body to move and cause trauma in another area of the
eye.
Test-Taking Strategy: Use knowledge of general principles of eye safety to answer this question.
Remember also that options that are similar are not likely to be correct. With this in mind,
eliminate options 3 and 4 first. Choose between the remaining options knowing that the correct
answer is the one that provides relief to the client without causing further harm. Review the
measures to treat eye injuries if you had difficulty with this question.
Level of Cognitive Ability: Application
Client Needs: Physiological Integrity
Integrated Process: Nursing Process/Implementation
Content Area: Child Health
Reference: Wong, D., & Hockenberry, M. (2003). Nursing care of infants and children (7th ed.).
St. Louis: Mosby, p. 1003.

1236. After a routine eye examination, a client has been told there are refractive errors in both
eyes. The nurse explains to the client that this problem is primarily treated by:
1. Eye drops
2. Rigid contact lenses
3. Prescription of corrective lenses
4. Keratoplasty
Answer: 3
Rationale: Errors of refraction in vision include astigmatism, presbyopia, myopia, and
hyperopia. Corrective lenses, or eyeglasses, are the most common method used to correct errors
of refraction. Eye drops would be used for several eye conditions, most commonly glaucoma.
The client may or may not need rigid contact lenses, and this is not the most common treatment.
A keratoplasty is a surgical procedure for cataracts.
Test-Taking Strategy: The issue of the question is the primary treatment for errors of refraction.
To answer this question correctly, you must understand the nature of this problem. With this in
mind, you could then eliminate options 1 and 4 easily. From the remaining options, eliminate
option 2 because of the word “rigid.” Rigid lenses may or may not be required. Many clients
wear soft contact lenses to correct errors of vision. Review care to the client with a refractive
error if you had difficulty with this question.
PN~Comp~Review-CD-1201-1300 19

Level of Cognitive Ability: Application


Client Needs: Health Promotion and Maintenance
Integrated Process: Nursing Process/Implementation
Content Area: Adult Health/Eye
Reference: Christensen, B., & Kockrow, E. (2003). Adult health nursing (4th ed.). St. Louis:
Mosby, pp. 364-365.

1237. A nurse has reinforced discharge teaching to a client following right eye cataract surgery
about ways to avoid strain on the operative eye. The nurse determines that the client needs
further instruction if the client makes which of the following statements?
1. “I can’t lift more than 5 pounds.”
2. “I should take stool softeners to prevent straining.”
3. “I can lie on my right side.”
4. “I cannot rub my eye.”
Answer: 3
Rationale: The client should not lie on the operative side in order to reduce strain on the surgical
eye. The information contained in options 1, 2, and 4 is correct, and indicates proper
understanding by the client of postoperative restrictions.
Test-Taking Strategy: Use the process of elimination and note the key words needs further
instruction. These words indicate a false response question and that you need to select the
incorrect client statement. Noting that the client had surgery on the right eye will direct you to
option 3. Review client instructions following cataract surgery if you had difficulty with this
question.
Level of Cognitive Ability: Analysis
Client Needs: Health Promotion and Maintenance
Integrated Process: Teaching/Learning
Content Area: Adult Health/Eye
Reference: Christensen, B., & Kockrow, E. (2003). Adult health nursing (4th ed.). St. Louis:
Mosby, p. 571.

1238. A client has an endocrine system dysfunction of the pancreas. The nurse anticipates that
the client will exhibit impaired secretion of which of the following substances?
1. Amylase
2. Lipase
3. Trypsin
4. Insulin
Answer: 4
Rationale: The pancreas produces both endocrine and exocrine secretions as part of its normal
function. The organ secretes insulin as a key endocrine hormone to regulate the blood glucose
level. Other pancreatic endocrine hormones are glucagon and somatostatin. The exocrine
pancreas produces digestive enzymes such as amylase, lipase, and trypsin.
Test-Taking Strategy: Use the process of elimination. Recall that the pancreas produces both
endocrine and exocrine secretions. Focusing on the key word endocrine in the question will
direct you to option 4. If this question was difficult, review the functions of the pancreas.
Level of Cognitive Ability: Comprehension
Client Needs: Physiological Integrity
PN~Comp~Review-CD-1201-1300 20

Integrated Process: Nursing Process/Data Collection


Content Area: Adult Health/Endocrine
Reference: Christensen, B., & Kockrow, E. (2003). Adult health nursing (4th ed.). St. Louis:
Mosby, pp. 174-175, 454-455.

1239. A nursing student is caring for a child with increased intracranial pressure. On review of
the chart, the student nurse notes that a transtentorial herniation has occurred. A nursing
instructor asks the student about this type of herniation. Which statement by the student
indicates a need for further research about this condition?
1. “The brain herniates downwards and around the tentorium cerebelli.”
2. “The herniation can be unilateral or bilateral.”
3. “It involves only anterior portions of the brain.”
4. “It can cause death if large amounts of tissue are involved.”
Answer: 3
Rationale: Transtentorial herniation occurs when part of the brain herniates downward and
around the tentorium cerebelli. It can be unilateral or bilateral and may involve anterior or
posterior portions of the brain. If a large amount of tissue is involved, it can cause death because
vital brain structures are compressed and become unable to perform their function.
Test-Taking Strategy: Note the key words a need for further research in the stem of the question.
These words indicate a false response question and that you need to select the incorrect student
statement. Noting the absolute word “only” in option 3 will direct you to this option. Review
this disorder if you had difficulty with this question.
Level of Cognitive Ability: Comprehension
Client Needs: Physiological Integrity
Integrated Process: Teaching/Learning
Content Area: Child Health
References: McKinney, E., James, S., Murray, S., & Ashwill, J. (2005). Maternal-child nursing
(2nd ed.). St. Louis: Elsevier, p. 1497.
Wong, D., & Hockenberry, M. (2003). Nursing care of infants and children (7th ed.). St. Louis:
Mosby, p. 1646.

1240. A 1-year old infant is admitted to the hospital for control of tonic-clonic seizures. The
nurse would avoid doing which of the following to protect the infant from injury?
1. Keep a padded tongue blade at the bedside for use during a seizure
2. Refrain from giving the infant toys that have bright blinking lights
3. Keep side rails and other hard objects padded
4. Turn the client to the side during a seizure
Answer: 1
Rationale: Attempting to place something in an infant’s mouth during a seizure in not helpful
even if it is padded. The mouth is usually clenched, and one would have to use force to open the
mouth. One must attempt to keep the airway clear and can do that by positioning (option 4).
Option 2 may be helpful in preventing a seizure, while option 3 safeguards the client’s physical
safety.
Test-Taking Strategy: The key word in the stem of the question is avoids. With this in mind,
eliminate options 3 and 4, which are obviously helpful actions. Choose between the remaining
options knowing either that tongue blades can be dangerous or that avoiding toys with lights
PN~Comp~Review-CD-1201-1300 21

would be a helpful item. Review care to the infant with seizures if you had difficulty with this
question.
Level of Cognitive Ability: Application

Client Needs: Safe, Effective Care Environment


Integrated Process: Nursing Process/Implementation
Content Area: Child Health
Reference: Price, D., & Gwin, J. (2005). Thompson’s pediatric nursing (9th ed.). Philadelphia:
W.B. Saunders, p. 242.

1241. A client receiving total parenteral nutrition (TPN) has a history of congestive heart failure.
The physician has ordered furosemide (Lasix) 40 mg orally daily to prevent fluid overload. The
nurse monitors which laboratory value to identify an adverse effect from this medication?
1. Glucose level
2. Sodium level
3. Potassium level
4. Magnesium level
Answer: 3
Rationale: Furosemide is a non–potassium-sparing diuretic, and insufficient replacement of
potassium may lead to hypokalemia. Although the glucose, sodium, and magnesium levels may
be monitored, these laboratory values are not specific to administering furosemide.
Test-Taking Strategy: Use the process of elimination. Recalling that furosemide is a non–
potassium-sparing diuretic will direct you to option 3. Review this medication if you had
difficulty with this question.
Level of Cognitive Ability: Analysis
Client Needs: Physiological Integrity
Integrated Process: Nursing Process/Data Collection
Content Area: Adult Health/Cardiovascular
Reference: Hodgson, B., & Kizior, R. (2005). Saunders nursing drug handbook 2005.
Philadelphia: W.B. Saunders, p. 480.

1242. A newborn infant has coarctation of the aorta (COA). The nurse would expect to note
which of the following data about an infant with this condition?
1. Cool upper extremities
2. Hepatomegaly
3. Blood pressure lower in the upper extremities and higher in the lower extremities
4. Bounding radial pulses, absent or weak femoral and pedal pulses
Answer: 4
Rationale: When there is narrowing within the aorta, there is increased pressure proximal to the
defect and decreased pressure distal to it. Therefore one would expect bounding pulses in the
arms and weak or absent pulses in the femoral and/or pedal areas. With decreased blood supply
to the lower extremities, that area would be cool to touch. The upper extremities would be
warm. The other options are incorrect.
Test-Taking Strategy: Use the process of elimination. Review each of the options listed,
anticipating the effects of coarctation on the circulatory dynamics of the infant. Eliminate
options 1 and 3 first, which are the opposite of the expected findings. Hepatomegaly is an
PN~Comp~Review-CD-1201-1300 22

associated finding if the client goes into heart failure as a complication of COA. With this in
mind, eliminate option 2. Review the findings in this disorder if you had difficulty with this
question.
Level of Cognitive Ability: Analysis
Client Needs: Physiological Integrity
Integrated Process: Nursing Process/Data Collection
Content Area: Maternity/Postpartum
Reference: McKinney, E., James, S., Murray, S., & Ashwill, J. (2005). Maternal-child nursing
(2nd ed.). St. Louis: Elsevier, pp. 1274-1275.

1243. A nurse is reviewing a chart of a child with a head injury. The nurse notes that the level of
consciousness has been documented as obtunded. Which of the following would the nurse
expect to note on data collection of the child?
1. Awake, alert, interacting with the environment
2. The ability to think clearly and rapidly is lost
3. The ability to recognize place or person is lost
4. Sleeps unless aroused and once aroused has limited interaction with the environment
Answer: 4
Rationale: Obtunded indicates that the child sleeps unless aroused and once aroused has limited
interaction with the environment. Full consciousness indicates that the child is alert, awake,
orientated, and interacts with the environment. Confusion indicates that the ability to think
clearly and rapidly is lost, and disorientation indicates that the ability to recognize place or
person is lost.
Test-Taking Strategy: Use the process of elimination. Remember, obtunded indicates that the
child sleeps unless aroused and once aroused has limited interaction with the environment.
Review these data collection findings if you are unfamiliar with them.
Level of Cognitive Ability: Analysis
Client Needs: Physiological Integrity
Integrated Process: Nursing Process/Data Collection
Content Area: Child Health
Reference: Wong, D., Perry, S., & Hockenberry, M. (2002). Maternal child nursing care (2nd
ed.). St. Louis: Mosby, p. 1648.

1244. A nurse is monitoring a child with a head injury. On data collection, the nurse notes an
abnormal flexion of the upper extremities and an extension of the lower extremities. The nurse
documents that the child is experiencing:
1. Decorticate posturing
2. Decerebrate posturing
3. Flexion of the arms and legs
4. A normal expected positioning following head injury
Answer: 1
Rationale: Decorticate posturing is an abnormal flexion of the upper extremities, and an
extension of the lower extremities with possible plantar flexion of the feet. Decerebrate
posturing is an abnormal extension of the upper extremities with internal rotation of the upper
arms and wrists, and an extension of the lower extremities with some internal rotation.
PN~Comp~Review-CD-1201-1300 23

Test-Taking Strategy: Use the process of elimination. Option 4 can be easily eliminated first.
Next, eliminate option 3 because it is an incomplete description of the nurse’s observation of the
child. From the remaining options, it is necessary to know the findings that occur with either
decorticate or decerebrate posturing. If you had difficulty with this question, review head
injuries and posturing.
Level of Cognitive Ability: Analysis
Client Needs: Physiological Integrity
Integrated Process: Communication and Documentation
Content Area: Child Health
Reference: Price, D., & Gwin, J. (2005). Thompson’s pediatric nursing (9th ed.). Philadelphia:
W.B. Saunders, pp. 202-203.

1245. An older client complains of chronic constipation. The nurse instructs the client to:
1. Include rice and bananas in the diet
2. Increase the intake of sugar-free products
3. Increase fluid intake to at least eight glasses a day and increase dietary fiber
4. Increase potassium in the diet
Answer: 3
Rationale: Increase of fluid intake and dietary fiber will help to change the consistency of the
stool and make it easier for the client to pass. Increasing the intake of rice and bananas will
increase constipation. Increasing sugar-free products and potassium in the diet will not be
beneficial to the client.
Test-Taking Strategy: Use the process of elimination and knowledge of the physiological causes
of constipation and factors that will alter consistency of stool. Remember, an increase of fluid
intake and dietary fiber will help to change the consistency of the stool and make it easier for the
client to pass. If you had difficulty with this question, review the interventions for constipation.
Level of Cognitive Ability: Application
Client Needs: Health Promotion and Maintenance
Integrated Process: Teaching/Learning
Content Area: Fundamental Skills
Reference: Wold, G. (2004). Basic geriatric nursing (3rd ed.). St. Louis: Mosby, p. 244.

1246. A nurse has reinforced instructions to a new mother about how to perform postpartum
exercises. The nurse determines that the client understood the instructions if she states that:
1. She should alternately contract and relax the muscles of the perineal area
2. The use of postpartum exercises can result in stress urinary incontinence
3. Exercise should be delayed for 4 weeks to allow healing time
4. Strenuous exercises should be started while in the hospital
Answer: 1
Rationale: Kegel exercises are extremely important to strengthen the muscle tone of the perineal
area. Postpartum exercises can begin soon after birth. The initial exercises should be simple
with progression to increasingly strenuous exercises. Women who maintain the perineal muscle
tone may benefit in later life by the development of less stress urinary incontinence.
Test-Taking Strategy: Use the process of elimination and note the key words understood the
instructions. Focusing on the issue—postpartum exercises—will direct you to option 1. Review
this content if you had difficulty with this question.
PN~Comp~Review-CD-1201-1300 24

Level of Cognitive Ability: Comprehension


Client Needs: Physiological Integrity
Integrated Process: Nursing Process/Evaluation
Content Area: Maternity/Postpartum
Reference: Leifer, G. (2003). Introduction to maternity & pediatric nursing (4th ed.).
Philadelphia: W.B. Saunders, pp. 210-211.

1247. A nurse is preparing to care for a child with a head injury. On review of the records, the
nurse notes that the physician has documented decorticate posturing. The nurse plans care
knowing that this type of posturing indicates which of the following?
1. Damage to the midbrain
2. Damage to the pons
3. Damage to the diencephalon
4. A lesion in the cerebral hemisphere
Answer: 4
Rationale: Decorticate posturing indicates a lesion in the cerebral hemisphere or disruption of
the corticospinal tracts. Decerebrate posturing indicates damage in the diencephalon, midbrain,
or pons.
Test-Taking Strategy: Specific knowledge regarding the assessment findings related to head
injuries is required to answer this question. Remember, decorticate posturing indicates a lesion
in the cerebral hemisphere or disruption of the corticospinal tracts. If you are unfamiliar with
decorticate and decerebrate posturing, review this content.
Level of Cognitive Ability: Analysis
Client Needs: Physiological Integrity
Integrated Process: Nursing Process/Planning
Content Area: Child Health
Reference: Leifer, G. (2003). Introduction to maternity & pediatric nursing (4th ed.).
Philadelphia: W.B. Saunders, p. 560.

1248. A nurse is reviewing the record of a newborn infant in the nursery and notes that the
physician has documented the presence of a cephalohematoma. Based on this documentation,
the nurse expects to note which of the following on data collection of the infant?
1. Swelling of the soft tissues of the head and scalp
2. Edema resulting from bleeding below the periosteum of the cranium
3. A suture split greater than 1 cm
4. A hard, rigid immobile suture line
Answer: 2
Rationale: A cephalohematoma indicates edema resulting from bleeding below the periosteum of
the cranium. It does not cross the suture line. It is most likely due to ruptured blood vessels
from head trauma during birth. It develops within 24 to 48 hours after birth and may take 2 to 3
weeks to resolve. Option 1 identifies a caput succedaneum. Option 3 may indicate increased
intracranial pressure. Option 4 may be associated with premature closure or craniosynostosis
and should be investigated further.
Test-Taking Strategy: Use the process of elimination. Note the relationship between
cephalohematoma in the question and bleeding in the correct option. Review the characteristics
of a cephalohematoma if you had difficulty with this question.
PN~Comp~Review-CD-1201-1300 25

Level of Cognitive Ability: Analysis


Client Needs: Physiological Integrity
Integrated Process: Nursing Process/Data Collection
Content Area: Maternity/Postpartum
Reference: Price, D., & Gwin, J. (2005). Thompson’s pediatric nursing (9th ed.). Philadelphia:
W.B. Saunders, p. 49.

1249. A nurse employed in a well-baby clinic is collecting data on the language and
communication developmental milestones of a 4-month-old infant. Based on the age of the
infant, the nurse expects to note which highest level of developmental milestones?
1. Use of gestures
2. Babbling sounds
3. Cooing sounds
4. Increased interest in sounds
Answer: 2
Rationale: Babbling sounds are common between the ages of 3 and 4 months. Additionally, at
this age crying becomes more differentiated. Between the ages of 1 and 3 months, the infant will
produce cooing sounds. An increased interest in sounds occurs between 6 and 8 months, and the
use of gestures occurs between 9 and 12 months.
Test-Taking Strategy: Use the process of elimination. Noting the age of the infant will assist in
eliminating options 1 and 4. From the remaining options, focus on the age to direct you to option
2, because cooing occurs very early in infancy. Review developmental milestones if you had
difficulty with this question.
Level of Cognitive Ability: Comprehension
Client Needs: Health Promotion and Maintenance
Integrated Process: Nursing Process/Data Collection
Content Area: Child Health
Reference: Leifer, G. (2003). Introduction to maternity & pediatric nursing (4th ed.).
Philadelphia: W.B. Saunders, p. 390.

1250. A nurse employed in a well-baby clinic is providing nutrition instructions to a mother of a


1-month-old infant. The nurse instructs the mother:
1. To offer rice cereal mixed with breast milk or formula
2. To introduce strained vegetables one at a time
3. To introduce strained fruits one at a time
4. That breast milk or formula is the main food
Answer: 4
Rationale: Breast milk or formula is the main food throughout infancy. Rice cereal mixed with
breast milk or formula is introduced at 4 months of age. Strained vegetables, fruits, and meats,
are introduced one at a time and can begin at 6 months of age.
Test-Taking Strategy: Use the process of elimination. Focus on the age of the infant. Note the
similarity in options 1, 2, and 3. These three options address specific food items. Review age-
appropriate nutrition measures if you had difficulty with this question.
Level of Cognitive Ability: Application
Client Needs: Health Promotion and Maintenance
Integrated Process: Teaching/Learning
PN~Comp~Review-CD-1201-1300 26

Content Area: Child Health


Reference: Leifer, G. (2003). Introduction to maternity & pediatric nursing (4th ed.).
Philadelphia: W.B. Saunders, p. 375.

<AQ>1251. {PLACE FIGURE HERE (Fig. 4)}


Black, J., & Hawks, J., (2005). Medical-surgical nursing: Clinical
management for positive outcomes (7th ed.). Philadelphia: W.B. Saunders,
p. 2335
A nurse is collecting data from a client with rheumatoid arthritis. The nurse inspects the client’s
hands and notes these characteristic deformities. The nurse identifies this deformity as:
1. Swan neck deformity
2. Rheumatoid nodules
3. Boutonnière deformity
4. Ulnar drift
Answer: 4
Rationale: All of the conditions identified in the options can occur in rheumatoid arthritis. Ulnar
drift occurs when synovitis stretches and damages the tendons and eventually the tendons
become shortened and fixed. This damage causes subluxation (drift) of the joints.
Test-Taking Strategy: Use the process of elimination. Note the relationship between the
characteristics in the client’s hands and option 4—ulnar drift. Review the deformities that occur
in rheumatoid arthritis if you had difficulty with this question.
Level of Cognitive Ability: Analysis
Client Needs: Physiological Integrity
Integrated Process: Nursing Process/Data Collection
Content Area: Adult Health/Immune
Reference: Black, J., & Hawks, J. (2005). Medical-surgical nursing: Clinical management for
positive outcomes (7th ed.). Philadelphia: W.B. Saunders, p. 2335.

1252. A pediatric nurse is caring for a hospitalized toddler. The nurse determines that the most
appropriate play activity for the toddler is which of the following?
1. Playing with a push-pull toy
2. Playing peek-a-boo
3. Hand sewing a picture

4. Listening to music
Answer: 1
Rationale: The toddler has increased use of motor skills and enjoys manipulating small objects
such as toy people, cars, and animals. Push-pull toys are appropriate for this age. Option 2 is
most appropriate for an infant. Option 3 is most appropriate for a school-age child. Option 4 is
most appropriate for an adolescent.
Test-Taking Strategy: Note the age group of the child and think about the related developmental
stage to answer the question. Option 2 can be eliminated because this activity is most
appropriate for an infant. Next, eliminate option 4 knowing that this activity is most appropriate
for an adolescent. From the remaining options, recalling that play activities should meet the
need of the toddler’s increased use of motor skills will assist in directing you to option 1.
Review age-related activities and toys if you had difficulty with this question.
PN~Comp~Review-CD-1201-1300 27

Level of Cognitive Ability: Comprehension


Client Needs: Health Promotion and Maintenance
Integrated Process: Nursing Process/Planning
Content Area: Child Health
Reference: Leifer, G. (2003). Introduction to maternity & pediatric nursing (4th ed.).
Philadelphia: W.B. Saunders, p. 420.

1253. A nurse is admitting to the hospital a child with a diagnosis of lactose intolerance. Which
finding would the nurse expect to obtain on data collection?
1. Reports of frothy stools

2. Reports of profuse, watery diarrhea and vomiting

3. Reports of foul-smelling ribbon stools


4. Reports of diffuse abdominal pain unrelated to meals or activity
Answer: 1
Rationale: Lactose intolerance causes frothy stools. Abdominal distention, crampy abdominal
pain, and excessive flatus may also occur. Option 2 is a clinical manifestation of celiac disease.
Option 3 is a clinical manifestation of Hirschsprung’s disease. Option 4 is a clinical
manifestation of irritable bowel syndrome.
Test-Taking Strategy: Use the process of elimination. Focus on the diagnosis and recall the
clinical manifestations associated with lactose intolerance to direct you to option 1. Review the
clinical manifestations associated with this disorder if you had difficulty with this question.
Level of Cognitive Ability: Comprehension
Client Needs: Physiological Integrity
Integrated Process: Nursing Process/Data Collection
Content Area: Child Health
Reference: Leifer, G. (2003). Introduction to maternity & pediatric nursing (4th ed.).
Philadelphia: W.B. Saunders, p. 65.

1254. A nurse has provided dietary instructions to the mother of a child with celiac disease. The
nurse determines that further instructions are needed if the mother states that she will include
which of the following in the child’s nutritional plan?
1. Rice
2. Oatmeal
3. Corn
4. Vitamin supplements
Answer: 2
Rationale: Dietary management is the mainstay of treatment for the child with celiac disease.
All wheat, rye, barley, and oats should be eliminated from the diet and replaced with corn and
rice. Vitamin supplements, especially fat-soluble vitamins and folate, may be needed in the early
period of treatment to correct deficiencies.
Test-Taking Strategy: Note the key words further instructions are needed. These words indicate
a false response question and that you need to select the incorrect statement by the mother.
Recalling that wheat, rye, barley, and oats need to be eliminated from the diet will direct you to
PN~Comp~Review-CD-1201-1300 28

option 2. Review the diet for the child with celiac disease if you had difficulty with this
question.
Level of Cognitive Ability: Comprehension
Client Needs: Health Promotion and Maintenance
Integrated Process: Teaching/Learning
Content Area: Child Health
Reference: Price, D., & Gwin, J. (2005). Thompson’s pediatric nursing (9th ed.). Philadelphia:
W.B. Saunders, pp. 237-238.

1255. A nurse is admitting a newborn infant to the nursery and notes that the physician has
documented that the newborn has an omphalocele. The nurse collects data knowing that in this
condition, the viscera are:
1. Inside the abdominal cavity and under the dermis
2. Inside the abdominal cavity and under the skin
3. Outside the abdominal cavity but inside a translucent sac covered with peritoneum and
amniotic membrane
4. Outside the abdominal cavity and not covered with a sac
Answer: 3
Rationale: Omphalocele is an abdominal wall defect. It involves a large herniation of the gut
into the umbilical cord. The viscera are outside the abdominal cavity but inside a translucent sac
covered with peritoneum and amniotic membrane. Option 4 describes a gastroschisis. Options 1
and 2 describe an umbilical hernia.
Test-Taking Strategy: Use the process of elimination. Eliminate options 1 and 2 first because
they are similar. Knowledge that with an omphalocele the viscera are outside the abdominal
cavity and inside a sac will direct you to option 3. Review this disorder if you are unfamiliar
with it.
Level of Cognitive Ability: Comprehension
Client Needs: Physiological Integrity
Integrated Process: Nursing Process/Data Collection
Content Area: Maternity/Postpartum
Reference: Price, D., & Gwin, J. (2005). Thompson’s pediatric nursing (9th ed.). Philadelphia:
W.B. Saunders, p. 98.

1256. A child is diagnosed with intussusception. The nurse collects data on the child knowing
that which of the following is a characteristic of this disorder?
1. Incomplete development of the anus
2. Invagination of a section of the intestine into the distal bowel
3. The infrequent and difficult passage of dry stools
4. The presence of fecal incontinence
Answer: 2
Rationale: Intussusception is an invagination of a section of the intestine into the distal bowel. It
is the most common cause of bowel obstruction in children age 3 months to 6 years. Option 1
describes imperforate anus, and this disorder is diagnosed in the neonatal period. Option 3
describes constipation. Option 4 describes encopresis. Constipation can affect any child at any
time although it peaks at age 2 to 3 years. Encopresis generally affects preschool and school-
aged children.
PN~Comp~Review-CD-1201-1300 29

Test-Taking Strategy: Use the process of elimination. Options 3 and 4 can be eliminated first.
Focusing on the word “child” in the question should assist in eliminating option 1 because an
imperforate anus is diagnosed in the neonatal period. Review the characteristics of
intussusception if you had difficulty with this question.
Level of Cognitive Ability: Comprehension
Client Needs: Physiological Integrity
Integrated Process: Nursing Process/Data Collection
Content Area: Maternity/Postpartum
Reference: Price, D., & Gwin, J. (2005). Thompson’s pediatric nursing (9th ed.). Philadelphia:
W.B. Saunders, pp. 154-155.

1257. The administration of mineral oil has been prescribed for the child with constipation. The
nurse provides instructions to the mother regarding the administration of the mineral oil. Which
statement by the mother indicates effective teaching?
1. “I will administer the mineral oil followed by a glass of warm water.”
2. “I will mix the mineral oil with 8 ounces of warm juice prior to administration.”
3. “I will mix the mineral oil with chilled chocolate milk prior to administration.”
4. “I will administer the mineral oil prior to each meal.”
Answer: 3
Rationale: Mineral oil is best tolerated when it is given chilled or mixed with cold drinks.
Mixing the oil with chocolate milk, blending it with ice cubes and fruit juice, or chilling it helps
to disguise the taste.
Test-Taking Strategy: Use the process of elimination. Note the key words indicates effective
teaching. Eliminate options 1 and 2 because they are similar and indicate administering the
mineral oil with a warm fluid. From the remaining options, eliminate option 4 because
administering the mineral oil to a child before a meal will affect the child’s appetite and desire to
eat. Review the client teaching points related to the administration of mineral oil if you had
difficulty with this question.
Level of Cognitive Ability: Comprehension
Client Needs: Health Promotion and Maintenance

Integrated Process: Teaching/Learning


Content Area: Child Health
Reference: McKinney, E., James, S., Murray, S., & Ashwill, J. (2005). Maternal-child nursing
(2nd ed.). St. Louis: Elsevier, pp. 1123-1124.

<AQ>1258. {PLACE FIGURE HERE (Fig. 5)}


Black, J., & Hawks, J., (2005). Medical-surgical nursing: Clinical
management for positive outcomes (7th ed.). Philadelphia: W.B. Saunders, p. 1443.
PN~Comp~Review-CD-1201-1300 30

An adult client is admitted to the emergency department following a burn injury. The burn

initially affected the client’s upper half of the anterior torso, and there were circumferential burns

to the lower half of both of the arms. The client’s clothes caught on fire and the client ran,

causing subsequent burn injuries to the entire face (anterior half of the head) and the upper half

of the posterior torso. Using the Rule of Nines, what would be the percent of the burn injury?

Answer: 31.5%
Rationale: According to the Rule of Nines, with the initial burn the upper half of the anterior
torso equals 9% and the lower half of both arms equals 9%. The subsequent burn included the
anterior half of the head equaling 4.5% and the upper half of the posterior torso equaling 9%.
This totals 31.5.
Test-Taking Strategy: Use the Rule of Nines to answer this question. The entire head equals 9%,
each entire arm equals 9% (both arms 18%), anterior or posterior torso each equals 18% (36%
for entire torso), each entire leg equals 18% (both legs 36%), perineum equals 1%. Remember:
9 (head), 18 (arms), 36 (torso), 36 (legs), 1 (perineum)—equaling 100. If you had difficulty with
this question, learn the Rule of Nines.
Level of Cognitive Ability: Analysis
Client Needs: Physiological Integrity
Integrated Process: Nursing Process/Data Collection
Content Area: Adult Health/Integumentary
Reference: Black, J., & Hawks, J. (2005). Medical-surgical nursing: Clinical management for
positive outcomes (7th ed.). Philadelphia: W.B. Saunders, p. 1443.

1259. A nurse is caring for a 1-year-old child following cleft palate repair. Following feeding,
the nurse plans which appropriate action?
1. Rinsing the mouth with water
2. Cleaning the mouth with diluted hydrogen peroxide
3. Using cotton swabs saturated with half-strength Betadine solution to clean the mouth
4. Using a soft lemon and glycerin swab to clean the mouth
Answer: 1
Rationale: Following cleft palate repair, the mouth is rinsed with water after feedings to clean
the palate repair site. Rinsing food and residual sugars from the suture line reduces the risk of
infection. Options 2, 3, and 4 are incorrect procedures, and the solutions identified in these
options should not be used.
Test-Taking Strategy: Consider the anatomical location of the surgical site. Note that the types
of solutions identified in options 2, 3, and 4 can cause damage to the suture site. This focus
should direct you to option 1. Review care to a child following cleft palate repair if you had
difficulty with this question.
Level of Cognitive Ability: Application
Client Needs: Physiological Integrity
Integrated Process: Nursing Process/Planning
Content Area: Child Health
PN~Comp~Review-CD-1201-1300 31

Reference: Leifer, G. (2003). Introduction to maternity & pediatric nursing (4th ed.).
Philadelphia: W.B. Saunders, p. 329.

1260. A newborn infant is diagnosed with gastroesophageal reflux (GER). The mother of the
infant asks the nurse to explain the diagnosis. The nurse plans to base the response on which
description of this disorder?
1. A portion of the stomach protrudes through the esophageal hiatus of the diaphragm
2. Abdominal contents herniate through an opening of the diaphragm
3. Gastric contents regurgitate back into the esophagus
4. The esophagus terminates before it reaches the stomach
Answer: 3
Rationale: GER is regurgitation of gastric contents back into the esophagus. Option 1 describes
a hiatal hernia. Option 2 describes a congenital diaphragmatic hernia. Option 4 describes
esophageal atresia.
Test-Taking Strategy: Use the process of elimination. Note the relationship between the word
“reflux” in the name of the diagnosis and “regurgitate” in the correct option. Review the
characteristics of GER if you had difficulty with this question.
Level of Cognitive Ability: Comprehension
Client Needs: Physiological Integrity
Integrated Process: Nursing Process/Planning
Content Area: Maternity/Postpartum
Reference: Leifer, G. (2003). Introduction to maternity & pediatric nursing (4th ed.).
Philadelphia: W.B. Saunders, p. 665.

1261. A nurse is collecting data on a newborn infant with a diagnosis of hiatal hernia. Which
finding would the nurse most specifically expect to note in the infant?
1. Excessive oral secretions
2. Coughing, wheezing, and short periods of apnea
3. Bowel sounds heard over the chest
4. Hiccupping and spitting up after a meal
Answer: 2
Rationale: Clinical manifestations associated with hiatal hernia specifically include vomiting,
coughing, wheezing, short periods of apnea, and failure to thrive. Option 1 is a clinical
manifestation of esophageal atresia and tracheoesophageal fistula. Option 3 is a clinical
manifestation associated with congenital diaphragmatic hernia. Option 4 is a clinical
manifestation of gastroesophageal reflux.
Test-Taking Strategy: Use the process of elimination. Recalling that a protrusion of a portion of
the stomach through the esophageal hiatus of the diaphragm occurs in a hiatal hernia will direct
you to option 2. If you had difficulty with this question, review the clinical manifestations of
hiatal hernia.
Level of Cognitive Ability: Comprehension
Client Needs: Physiological Integrity
Integrated Process: Nursing Process/Data Collection
Content Area: Maternity/Postpartum
Reference: McKinney, E., James, S., Murray, S., & Ashwill, J. (2005). Maternal-child nursing
(2nd ed.). St. Louis: Elsevier, p. 1115.
PN~Comp~Review-CD-1201-1300 32

1262. A nurse is reviewing the laboratory results of an infant suspected of having hypertropic
pyloric stenosis. Which of the following would the nurse most likely expect to note in this
infant?
1. Respiratory acidosis
2. Respiratory alkalosis
3. Metabolic acidosis
4. Metabolic alkalosis
Answer: 4
Rationale: Laboratory findings in an infant with hypertropic pyloric stenosis include metabolic
alkalosis as a result of the vomiting that occurs in this disorder. Additional findings include
decreased serum potassium and sodium levels, increased blood pH, increased bicarbonate level,
and decreased chloride level.
Test-Taking Strategy: Use the process of elimination. Recalling the concepts related to acid-
base balance and the clinical manifestations—progressive, projectile, nonbilous vomiting—that
occur in hypertropic pyloric stenosis will direct you to option 4. Remember that metabolic
alkalosis occurs from vomiting. Review the clinical manifestations that occur in this disorder if
you had difficulty with this question.
Level of Cognitive Ability: Analysis
Client Needs: Physiological Integrity
Integrated Process: Nursing Process/Data Collection
Content Area: Child Health
Reference: Wong, D., Perry, S., & Hockenberry, M. (2002). Maternal child nursing care (2nd
ed.). St. Louis: Mosby, pp. 1446-1447.

1263. A client has had a midline episiotomy. In relation to clients with other types of
episiotomies, the nurse anticipates that the client will generally experience:
1. Greater blood loss
2. No blood loss
3. Greater pain
4. Less pain
Answer: 4
Rationale: Midline episiotomies are effective, easily repaired, and generally result in less pain.
The blood loss is greater and the repair is more difficult and painful with the mediolateral than
the midline episiotomy.
Test-Taking Strategy: Use the process of elimination to make your selection. Eliminate option 2
first because of the absolute word “no.” Eliminate options 1 and 3 next because of the presence
of the word “greater” in both options. Knowledge concerning the differences between midline
and mediolateral episiotomies allows for the elimination of the options indicating greater pain
and greater blood loss. Review the differences between these types of episiotomies if you had
difficulty with this question.
Level of Cognitive Ability: Comprehension
Client Needs: Physiological Integrity
Integrated Process: Nursing Process/Data Collection
Content Area: Maternity/Postpartum
PN~Comp~Review-CD-1201-1300 33

Reference: Leifer, G. (2005). Maternity nursing (9th ed.). Philadelphia: W.B. Saunders, pp. 245-
246.

1264. A client with a history of angina pectoris tells the nurse that chest pain usually occurs after
going up two flights of stairs or after walking four blocks. The nurse interprets that the client is
experiencing which of the following types of angina?
1. Stable
2. Unstable
3. Variant
4. Intractable
Answer: 1
Rationale: Stable angina is triggered by a predictable amount of effort or emotion. Unstable
angina is triggered by an unpredictable amount of exertion or emotion, and may occur at night;
the attacks increase in number, duration, and severity over time. Variant angina is triggered by
coronary artery spasm; the attacks are of longer duration than classic angina, and tend to occur
early in the day and at rest. Intractable angina is chronic and incapacitating, and is refractory to
medical therapy.
Test-Taking Strategy: Use the process of elimination. Noting that the question indicates the
presence of chest pain with a predictable amount of effort will direct you to option 1. Review the
characteristics of the various types of angina if you had difficulty with this question.
Level of Cognitive Ability: Comprehension
Client Needs: Physiological Integrity
Integrated Process: Nursing Process/Data Collection
Content Area: Adult Health/Cardiovascular
Reference: Black, J., & Hawks, J. (2005). Medical-surgical nursing: Clinical management for
positive outcomes (7th ed.). Philadelphia: W.B. Saunders, p. 1704.

1265. A nurse who plans to administer a prescribed oxytocic medication after delivery to
stimulate uterine contractions and prevent hemorrhage should prepare to give the medication
after delivery of the:
1. Infant’s body
2. Placenta
3. Infant’s head
4. Infant’s shoulders
Answer: 2
Rationale: Oxytocics are administered because they stimulate the uterus to contract, thereby
helping to prevent hemorrhage after the placenta is removed. If an oxytocic medication is
ordered, the nurse plans to administer the medication after the placenta has been expelled. If the
medication is given before the delivery of the placenta, it can cause the uterus to contract and
restrict delivery.
Test-Taking Strategy: Use the process of elimination. Remember that options that are similar
are not likely to be correct. In this case, each of the incorrect options addresses a part of the
infant’s body, while the correct option addresses the placenta. Review the procedure for
administering these medications if you had difficulty with this question.
Level of Cognitive Ability: Application
Client Needs: Physiological Integrity
PN~Comp~Review-CD-1201-1300 34

Integrated Process: Nursing Process/Planning


Content Area: Maternity/Intrapartum
References: Leifer, G. (2003). Introduction to maternity & pediatric nursing (4th ed.).
Philadelphia: W.B. Saunders, p. 206.
Murray, S., McKinney, E., & Gorrie, T. (2002). Foundations of maternal-newborn nursing (3rd
ed.). Philadelphia: W.B. Saunders, p. 443.

1266. A nurse is teaching a client with angina pectoris about disease management and lifestyle
changes that are necessary in order to control disease progression. Which statement by the client
indicates a need for further teaching?
1. “I will take nitroglycerin whenever chest discomfort begins.”
2. “I will use muscle relaxation to cope with stressful situations.”
3. “It is best to exercise once a week for an hour.”
4. “I will avoid using table salt with meals.”
Answer: 3
Rationale: Exercise is most effective when done at least 3 times a week for 20 to 30 minutes to
reach a target heart rate. Other healthy habits include limiting salt and fat in the diet and using
stress management techniques. The client should also be taught to take nitroglycerin before any
activity that causes pain, and to take the medication at the first sign of chest discomfort.
Test-Taking Strategy: Use the process of elimination, noting the key words need for further
teaching. These words indicate a false response question and that you need to select the
incorrect client statement. Options 1, 2, and 4 are appropriate client interventions and are
therefore eliminated. Remember, exercise should be done at least three times a week for optimal
benefit. Review teaching points for the client with angina pectoris if you had difficulty with this
question.
Level of Cognitive Ability: Comprehension
Client Needs: Health Promotion and Maintenance
Integrated Process: Teaching/Learning
Content Area: Adult Health/Cardiovascular

Reference: Christensen, B., & Kockrow, E. (2003). Adult health nursing (4th ed.). St. Louis:
Mosby, pp. 307-308.

1267. A nurse is caring for an older client at home whose family has gone out for the day.
During the visit, the client experiences chest pain that is unrelieved by three sublingual
nitroglycerin tablets given by the nurse. Which action by the nurse would be appropriate at this
time?
1. Notify a family member who is the next of kin
2. Inform the home care agency supervisor of the client's complaint
3. Call for an ambulance to transport the client to the emergency department
4. Drive the client to the physician’s office
Answer: 3
Rationale: Chest pain that is unrelieved by rest and 3 doses of nitroglycerin administered 5
minutes apart may not be typical anginal pain, but may signal myocardial infarction (MI). Since
the risk of sudden cardiac death is greatest in the first 24 hours after MI, it is imperative that the
client receive emergency cardiac care. A physician’s office may not be equipped to treat MI.
PN~Comp~Review-CD-1201-1300 35

Communication with the family or home care agency delays client treatment, which is needed
immediately.
Test-Taking Strategy: Focus on the data in the question. Eliminate options 1 and 2 first because
they delay emergency treatment. Since MI is a medical emergency, select option 3 from the
remaining options. Review care to the client experiencing an MI if you had difficulty with this
question.
Level of Cognitive Ability: Application
Client Needs: Physiological Integrity
Integrated Process: Nursing Process/Implementation
Content Area: Adult Health/Cardiovascular
References: Christensen, B., & Kockrow, E. (2003). Adult health nursing (4th ed.). St. Louis:
Mosby, p. 309.
Linton, A., & Maebius, N. (2003). Introduction to medical-surgical nursing (3rd ed.).
Philadelphia: W.B. Saunders, p. 583.

1268. A nurse is working with a client who has been diagnosed with Prinzmetal’s (variant)
angina. The nurse plans to reinforce to the client that this type of angina:
1. Is most effectively managed by beta-blocking agents
2. Is generally treated with calcium channel blocking agents
3. Has the same risk factors as stable and unstable angina
4. Improves with a low-sodium, high-potassium diet
Answer: 2
Rationale: Prinzmetal’s angina results from spasm of the coronary vessels. The risk factors are
unknown, and this type of angina is relatively unresponsive to nitrates. Beta-blockers are
contraindicated, because they may actually worsen the spasm. Diet therapy is not indicated.
Test-Taking Strategy: Use the process of elimination recalling that variant angina is a functional
disorder that results from coronary spasm and is not due to atherosclerosis. The correct option
therefore is one that is effective against spasm, which is option 2. Review the treatment for
Prinzmetal’s angina if you had difficulty with this question.
Level of Cognitive Ability: Application
Client Needs: Health Promotion and Maintenance
Integrated Process: Teaching/Learning
Content Area: Adult Health/Cardiovascular
Reference: Black, J., & Hawks, J. (2005). Medical-surgical nursing: Clinical management for
positive outcomes (7th ed.). Philadelphia: W.B. Saunders, pp. 1704-1705.

1269. A nurse working in a long-term care facility is collecting data from a client experiencing
chest pain. The nurse would interpret that the pain is most likely due to myocardial infarction
(MI) if which of the following observations is made by the nurse?
1. The client is not experiencing nausea or vomiting
2. The client says the pain began while trying to open a jammed dresser drawer
3. The pain has not been relieved by rest and by taking three nitroglycerin tablets
4. The pain is described as substernal and radiating to the left arm
Answer: 3
Rationale: The pain of angina may radiate to the left shoulder, arm, neck, or jaw. It is often
precipitated by exertion or stress, has few associated symptoms, and is relieved by rest and
PN~Comp~Review-CD-1201-1300 36

nitroglycerin. The pain of MI may also radiate to the left arm, shoulder, jaw, and neck. It
typically begins spontaneously, lasts longer than 30 minutes, and is frequently accompanied by
associated symptoms (such as nausea, vomiting, dyspnea, diaphoresis, anxiety). The pain of MI
is not relieved by rest and nitroglycerin, and requires opioid analgesics, such as morphine sulfate,
for relief.
Test-Taking Strategy: Use the process of elimination seeking the option that differentiates
anginal pain from that of MI. Recalling that a classic hallmark of the pain from MI is that it is
unrelieved by rest and nitroglycerin will direct you to option 3. Review the clinical
manifestations and characteristics of MI if you had difficulty with this question.
Level of Cognitive Ability: Comprehension
Client Needs: Physiological Integrity
Integrated Process: Nursing Process/Data Collection
Content Area: Adult Health/Cardiovascular
References: Christensen, B., & Kockrow, E. (2003). Adult health nursing (4th ed.). St. Louis:
Mosby, p. 308.
Linton, A., & Maebius, N. (2003). Introduction to medical-surgical nursing (3rd ed.).
Philadelphia: W.B. Saunders, p. 581.

1270. A nurse observes a client following childbirth for normal maternal physiological changes
that are anticipated. The nurse would document which expected change in vital signs?
1. Altered respiration
2. Increased pulse rate
3. Altered level of consciousness
4. Slowed pulse rate
Answer: 4
Rationale: Following delivery of the placenta, the maternal cardiac system begins to make
changes. Bradycardia is common with a range of 50 to 70 beats per minute.
Test-Taking Strategy: The key word in the question is expected. Use knowledge of maternal
changes following delivery and the process of elimination to answer the question. Remember, a
slowed pulse rate is an expected finding. Review these expected changes if you had difficulty
with this question.
Level of Cognitive Ability: Comprehension
Client Needs: Physiological Integrity
Integrated Process: Communication and Documentation
Content Area: Maternity/Antepartum
References: Leifer, G. (2005). Maternity nursing (9th ed.). Philadelphia: W.B. Saunders, pp.
196-197.
McKinney, E., James, S., Murray, S., & Ashwill, J. (2005). Maternal-child nursing (2nd ed.). St.
Louis: Elsevier, p. 477.

1271. A nurse assisting in the care of a woman in labor should focus primary attention on which

of the following at the time of delivery?

1. Mother
PN~Comp~Review-CD-1201-1300 37

2. Primary health care provider


3. Infant
4. Support person
Answer: 3
Rationale: The nurse’s primary responsibility at the time of delivery is focused on the infant.
The primary health care provider is primarily responsible for the care of the mother. The support
persons are responsible for themselves.
Test-Taking Strategy: Use the process of elimination. Noting the key words at the time of

delivery will assist in directing you to option 3. Review the nursing responsibilities following

delivery if you had difficulty with this question.

Level of Cognitive Ability: Application


Client Needs: Physiological Integrity
Integrated Process: Nursing Process/Implementation
Content Area: Maternity/Intrapartum
Reference: Leifer, G. (2005). Maternity nursing (9th ed.). Philadelphia: W.B. Saunders, p. 124.

1272. A client with no history of heart disease has experienced acute myocardial infarction and

has been given thrombolytic therapy with t-PA (tissue plasminogen activator). The nurse

interprets that the client is most likely experiencing a complication of this therapy if which of the

following occurs?

1. Tarry stools
2. Nausea and vomiting
3. Decreased urine output
4. Orange colored urine
Answer: 1
Rationale: Thrombolytic agents are used to dissolve existing thrombi, and the nurse must
monitor the client for obvious or occult signs of bleeding. This includes assessment for obvious
bleeding within the gastrointestinal (GI) tract, urinary system, and skin. It also includes
hematesting secretions for occult blood. Option 1 is the only option that indicates the presence
of blood.
Test-Taking Strategy: Use the process of elimination. Note the word “thrombolytic,” meaning
to dissolve clots, and focus your attention on blood coagulation. Look for the option that has a
hematological connection; in this case, option 1, which indicates bleeding from the GI tract.
Review nursing care for the client receiving thrombolytic therapy if you had difficulty with this
question.
Level of Cognitive Ability: Analysis
Client Needs: Physiological Integrity
Integrated Process: Nursing Process/Data Collection
PN~Comp~Review-CD-1201-1300 38

Content Area: Adult Health/Cardiovascular


Reference: Skidmore-Roth, L. (2005). Mosby’s drug guide for nurses (6th ed.). St. Louis:
Mosby, p. 31.

1273. A 15-year-old client who is pregnant and unwed says, “My life was unbearable before I
met Johnny. My mother beats me up every day and my dad has been sleeping with me since I
was 10 years old!” Which response is appropriate for the nurse to make?
1. “Why didn’t you just report your parents for abuse?”
2. “What are you saying? Your parents abused you so you got pregnant?”
3. “Sounds like you decided to have a baby so you’d have someone for yourself.”
4. “It seems that you needed help to separate from your family. Do you feel you are ready to
have a baby with Johnny?”
Answer: 4
Rationale: Adolescent pregnancy outside of marriage can arise from female low self-esteem,
fears of inadequacy, and desperation to escape from an abusive and dysfunctional family. The
most therapeutic response is the one that uses restatement and repeats the main thought that the
client expressed. This assures the client that the nurse is listening and is attempting to validate
what the client has said. Options 1, 2, and 3 are nontherapeutic. Option 1 reflects knowledge
deficit on the nurse’s part. Option 2 is insensitive and makes assumptions. Option 3 makes
connections that are assumed and imply judgmental bias.
Test-Taking Strategy: Use the process of elimination and therapeutic communication techniques
to direct you to option 4. Review these techniques if you had difficulty with this question.
Level of Cognitive Ability: Application
Client Needs: Psychosocial Integrity
Integrated Process: Communication and Documentation
Content Area: Mental Health
Reference: Morrison-Valfre, M. (2005). Foundations of mental health care (3rd ed.). St. Louis:
Mosby, p. 88.

1274. A nurse collecting data on a client during the second stage of labor notes a slowing of the
fetal heart rate (FHR) with a loss of variability, and determines that these are indicators of
possible complications. The nurse plans to immediately reposition the client to the:
1. Back and administer oxygen by mask at 8 to 10 L per minute
2. Side and administer oxygen by nasal cannula at 2 to 4 L per minute
3. Back and administer oxygen by nasal cannula at 2 to 4 L per minute
4. Side and administer oxygen by mask at 8 to 10 L per minute
Answer: 4
Rationale: Prompt treatment must be initiated when the FHR begins to slow or a loss of
variability is identified during fetus delivery. To facilitate oxygenation of the mother and fetus,
the mother is turned to her side to reduce uterine pressure on the ascending vena cava and
descending aorta. The greater flow rate for oxygen is also indicated.
Test-Taking Strategy: To answer this question correctly, you must determine both the correct
positioning and the correct amount of oxygen that are required by the client. Eliminate options 2
and 3 first, reasoning that higher levels of oxygen are better. From the remaining options, recall
that the client should be turned to the side. Review care to the client in labor if you had difficulty
with this question.
PN~Comp~Review-CD-1201-1300 39

Level of Cognitive Ability: Application


Client Needs: Physiological Integrity
Integrated Process: Nursing Process/Planning
Content Area: Maternity/Intrapartum
Reference: Leifer, G. (2005). Maternity nursing (9th ed.). Philadelphia: W.B. Saunders, p. 75.

1275. A client has had surgery to repair a fractured left hip. The nurse plans to use which of the
following most important items when repositioning the client from side to side in the bed?
1. Bed pillow
2. Overhead trapeze
3. Abductor splint
4. Adductor splint
Answer: 3
Rationale: Following surgery to repair a fractured hip, an abductor splint is used to maintain the
affected extremity in good alignment. An overhead trapeze and bed pillow are also used, but
they are not the priority item to be used in repositioning.
Test-Taking Strategy: Use the process of elimination focusing on the issue—repositioning.
Remember that an abductor splint is used to maintain correct alignment in a client following
repair of a fractured hip. Review care to the client following hip surgery if you had difficulty
with this question.
Level of Cognitive Ability: Application
Client Needs: Physiological Integrity
Integrated Process: Nursing Process/Planning
Content Area: Adult Health/Musculoskeletal
References: Black, J., & Hawks, J. (2005). Medical-surgical nursing: Clinical management for
positive outcomes (7th ed.). Philadelphia: W.B. Saunders, p. 589.
Christensen, B., & Kockrow, E. (2003). Adult health nursing (4th ed.). St. Louis: Mosby, p. 127.

1276. A client with a 4-day-old lumbar vertebral fracture is experiencing muscle spasms. The
nurse avoids using which of the following in an effort to relieve the spasm?
1. Prescribed intermittent traction
2. Analgesics
3. Heat
4. Cold
Answer: 4
Rationale: Traction, analgesics, and heat may all be used to relieve the pain of muscle spasm in
the client with a vertebral fracture. The use of ice is incorrect because ice is only applied to a site
for the first 24 hours after this type of injury. Application of ice to the spine of a client could be
uncomfortable and result in feelings of being chilled.
Test-Taking Strategy: Use the process of elimination and note the words “4-day-old” and
“avoids.” Option 2 can be easily eliminated first. Next, eliminate option 1 because of the word
“prescribed.” Use concepts related to the principles of heat and cold to select from the remaining
options. If this question was difficult, review care to the client with a vertebral fracture.
Level of Cognitive Ability: Application
Client Needs: Physiological Integrity
Integrated Process: Nursing Process/Implementation
PN~Comp~Review-CD-1201-1300 40

Content Area: Adult Health/Musculoskeletal


Reference: Christensen, B., & Kockrow, E. (2003). Adult health nursing (4th ed.). St. Louis:
Mosby, pp. 140-142.

1277. A nurse has reviewed activity restrictions with a client who is being discharged following
insertion of a femoral head prosthesis. The nurse determines that the client understands the
material presented if the client states to:
1. Use a raised toilet seat
2. Exercise the leg past the point of 90-degree flexion
3. Bend carefully to put on socks and shoes
4. Sit in chairs without arms for better mobility
Answer: 1
Rationale: The client who has had an insertion of a femoral head prosthesis should use a raised
toilet seat. The client should also maintain the leg in a neutral, straight position when lying,
sitting, or walking. The leg should not be adducted, internally rotated, or flexed more than 90
degrees. The client should sit in chairs that have arms so there will be assistance when the client
is ready to rise from the sitting position. The client should avoid putting on own socks and shoes
for 8 weeks after surgery because it would force the leg into acute flexion.
Test-Taking Strategy: Use the process of elimination, noting the words “understands the material
presented.” Answer the question by evaluating each of the options in terms of the risk for
prosthesis displacement that it carries. Review the activity restrictions for the client following
insertion of a femoral head prosthesis if you had difficulty with this question.
Level of Cognitive Ability: Comprehension
Client Needs: Health Promotion and Maintenance
Integrated Process: Nursing Process/Evaluation
Content Area: Adult Health/Musculoskeletal
Reference: Black, J., & Hawks, J. (2005). Medical-surgical nursing: Clinical management for
positive outcomes (7th ed.). Philadelphia: W.B. Saunders, p. 643.

1278. A nurse is talking to a client who underwent a below the knee amputation 2 days earlier.
The client says to the nurse, “I hate looking at this; I feel that I’m not even myself anymore.”
The nurse understands that the client is experiencing which problem?
1. Ineffective health maintenance
2. Self-care deficit
3. Disturbed body image
4. Ineffective coping
Answer: 3
Rationale: Disturbed body image is characterized by negative verbalizations or feelings about a
body part. This is a common response after amputation. The nurse supports the client and
assists the client to work through these feelings. The client may also have the other problems
listed in options 1, 2, and 4, but disturbed body image is the problem that correlates best with the
client statement.
Test-Taking Strategy: Use the process of elimination. In this question, focusing on the client’s
statement will direct you to option 3. Review care to the client following amputation if you had
difficulty with this question.
Level of Cognitive Ability: Analysis
PN~Comp~Review-CD-1201-1300 41

Client Needs: Psychosocial Integrity


Integrated Process: Nursing Process/Data Collection
Content Area: Adult Health/Musculoskeletal
Reference: Linton, A., & Maebius, N. (2003). Introduction to medical-surgical nursing (3rd
ed.). Philadelphia: W.B. Saunders, pp. 842, 845.

1279. A client receiving heparin sodium by continuous intravenous (IV) infusion removes the
tubing from the pump to change the hospital gown. The licensed practical nurse (LPN) is
concerned that the client received a bolus of medication. The LPN immediately notifies the
registered nurse and then checks to see if which medication is available in the medication cart?
1. Aminocaproic acid (Amicar)
2. Vitamin K (AquaMEPHYTON)
3. Enoxaparin (Lovenox)
4. Protamine sulfate
Answer: 4
Rationale: If the tubing is removed from an IV pump and the tubing is not clamped, the client
will receive a bolus of the solution and the medication contained in the solution. The client who
receives a bolus dose of heparin is at risk for bleeding. A partial thromboplastin time (PTT) will
be drawn and evaluated. If the results of the PTT are too high, a dose of protamine sulfate, the
antidote for heparin, may be prescribed. Amicar is an antithrombinolytic (inhibits clot
breakdown). Lovenox is an anticoagulant. Vitamin K is the antidote for warfarin sodium
(Coumadin).
Test-Taking Strategy: Use the process of elimination. Recalling that the antidote for heparin is
protamine sulfate will direct you to option 4. Review the medications identified in the options if
you had difficulty with this question.
Level of Cognitive Ability: Application
Client Needs: Physiological Integrity
Integrated Process: Nursing Process/Implementation
Content Area: Pharmacology
Reference: Gahart, B., & Nazareno, A. (2004). 2004 Intravenous medications (20th ed.). St.
Louis: Mosby, p. 615.

1280. A client returning to the medical nursing unit following cardiac catheterization has a stat
order to receive a dose of procainamide (Pronestyl). The licensed practical nurse assisting in
caring for the client obtains which of the following pieces of equipment to most adequately
determine the client’s response to this medication?
1. Cardiac monitor
2. Noninvasive blood pressure cuff
3. Pulse oximeter
4. Glucometer
Answer: 1
Rationale: Procainamide is an antidysrhythmic medication often used to treat ventricular
dysrhythmias that do not adequately respond to lidocaine (LidoPen). The effectiveness of this
medication is best determined by evaluating the client’s cardiac rhythm. Thus, a cardiac monitor
is of greatest value, although the blood pressure cuff and pulse oximeter will provide general
PN~Comp~Review-CD-1201-1300 42

information about the client’s cardiovascular status. A glucometer is not needed for this client
with the information presented.
Test-Taking Strategy: Use the process of elimination. Note the key words most adequately
determine. Recalling that procainamide is an antidysrhythmic will direct you to option 1.
Review care to the client receiving procainamide if you had difficulty with this question.
Level of Cognitive Ability: Analysis
Client Needs: Physiological Integrity
Integrated Process: Nursing Process/Evaluation
Content Area: Pharmacology
Reference: Hodgson, B., & Kizior, R. (2005). Saunders nursing drug handbook 2005.
Philadelphia: W.B. Saunders, p. 889.

1281. A 10-year-old female client who has been reported for drawing sexually explicit scenes in
her textbooks says to the psychiatric nurse, “I just felt like it.” Which response is therapeutic for
the nurse to make in order to assess abuse-related symptoms?
1. “I am concerned about you. Are you now or have you ever been abused?”
2. “Well, a picture paints a thousand words.”
3. “You just felt like destroying your textbooks?”
4. “Your parents and teachers are very concerned about your drawings.”
Answer: 1
Rationale: The behaviors that this child engaged in are a warning signal of distress. Option 1 is
the only option that specifically addresses abuse. In option 2 the nurse is insensitive, sarcastic,
and intrusive. In option 3 the nurse is assessing the client’s destructive behaviors, not the
possible sexual abuse history. In option 4 while the nurse is trying to assess the client’s abuse-
related symptoms, the nurse uses indirect means rather than straightforward expressions of the
nurse’s concern.
Test-Taking Strategy: Use the process of elimination and therapeutic communication techniques,
focusing on the issue of the question. Noting the key words assess abuse-related symptoms will
direct you to option 1. Review the assessment techniques related to possible abuse if you had
difficulty with this question.
Level of Cognitive Ability: Application
Client Needs: Psychosocial Integrity
Integrated Process: Communication and Documentation
Content Area: Mental Health
Reference: Morrison-Valfre, M. (2005). Foundations of mental health care (3rd ed.). St. Louis:
Mosby, p. 88.

1282. During a nursing interview a client says, “My daughter was murdered in her New York
apartment and her estranged husband called to tell me. I can’t stop myself from wondering if he
killed her.” Which of the following is the therapeutic nursing response?
1. “It feels terrible to lose a daughter. I’d have suspicions about him too.”
2. “Have you shared your concerns with the police?”
3. “I don’t think that you should blame yourself one little bit.”
4. “I agree. What do you want to bet he did it?”
Answer: 2
PN~Comp~Review-CD-1201-1300 43

Rationale: Option 2 addresses the issue of the client’s statement. Options 1 and 4 are statements
that identify the process of agreeing with the client. Option 3 is not directly related to the issue
of the client’s statement.
Test-Taking Strategy: Use the process of elimination. Options 1 and 4 are not therapeutic in that
they use agreeing, which is a premature action that negates a full evaluation of the facts. Option
3 is not directly related to the issue of the question. Review therapeutic communication
techniques if you had difficulty with this question.
Level of Cognitive Ability: Application
Client Needs: Psychosocial Integrity
Integrated Process: Communication and Documentation
Content Area: Mental Health
Reference: Morrison-Valfre, M. (2005). Foundations of mental health care (3rd ed.). St. Louis:
Mosby, p. 88.

1283. A nurse is gathering data from a pregnant client about potential maternal physiological
risk factors. The nurse would be sure to obtain data about which of the following at this time?
1. Self-care needs
2. Weight
3. Support systems
4. Life stress
Answer: 2
Rationale: Height and weight relationships are important factors to assess when determining
potential physiological risk factors. Weight gains of more than 30 pounds could place the
woman at a higher risk for cephalopelvic disproportion and the need for cesarean birth. Options
1, 3, and 4 are not directly related to physiological risk factors.
Test-Taking Strategy: Note the key word physiological. Use Maslow’s Hierarchy of Needs
theory. Option 2 directly addresses a physiological need. Review data collection of the pregnant
client if you had difficulty with this question.
Level of Cognitive Ability: Comprehension
Client Needs: Physiological Integrity
Integrated Process: Nursing Process/Data Collection
Content Area: Maternity/Antepartum
Reference: Leifer, G. (2005). Maternity nursing (9th ed.). Philadelphia: W.B. Saunders, pp. 58-
59.

1284. A nurse is discussing smoking cessation with a client diagnosed with coronary artery
disease (CAD). Which statement would the nurse make to the client to try to motivate the client
to quit smoking?
1. “If you quit now, your risk of cardiovascular disease will decrease to that of a non-smoker in
3 to 4 years.”
2. “Since most of the damage has already been done, it will be all right to cut down a little at a
time.”
3. “If you totally quit smoking right now, you can cut your cardiovascular risk to zero within a
year.”
4. “None of the cardiovascular effects are reversible, but quitting might prevent lung cancer.”
Answer: 1
PN~Comp~Review-CD-1201-1300 44

Rationale: The risks to the cardiovascular system from smoking are noncumulative and are not
permanent. Three to four years after cessation, a client’s cardiovascular risk is similar to that of a
person who never smoked. Therefore options 2, 3, and 4 are incorrect.
Test-Taking Strategy: Use the process of elimination. Eliminate options 3 and 4 first because of
the absolute words “totally” and “none.” Knowledge about the cumulative risk of smoking on
cardiovascular status is needed to select between options 1 and 2. Review the importance of
smoking cessation in a client with CAD if you had difficulty with this question.
Level of Cognitive Ability: Application
Client Needs: Health Promotion and Maintenance
Integrated Process: Teaching/Learning
Content Area: Adult Health/Cardiovascular
Reference: Christensen, B., & Kockrow, E. (2003). Adult health nursing (4th ed.). St. Louis:
Mosby, p. 308.

1285. A client with heart failure is scheduled to be discharged to home with digoxin (Lanoxin)
and furosemide (Lasix) as ongoing prescribed medications. The nurse teaches the client to report
which item that indicates that the medications are not having the intended effect?
1. Cough accompanied by other signs of respiratory tract infection
2. Sudden increase in appetite
3. Weight gain of 2 to 3 pounds in a few days
4. High urine output during the day
Answer: 3
Rationale: Clients with heart failure should immediately report weight gain, loss of appetite,
shortness of breath with activity, edema, persistent cough, and nocturia. An increase in daytime
voiding is expected while on diuretic therapy (Lasix). A cough resulting from respiratory tract
infection does not necessarily indicate that heart failure is exacerbating.
Test-Taking Strategy: Use the process of elimination. Note the key words not having the
intended effect. Eliminate option 1 first because it is related to respiratory tract infection and
does not indicate impending heart failure. Option 2 is eliminated next, because it is a positive
sign. Option 4 is expected with diuretic therapy administered in the morning. Sudden weight
gain would accompany fluid retention and should be reported. Review home care instructions
for the client with heart failure if you had difficulty with this question.
Level of Cognitive Ability: Analysis
Client Needs: Health Promotion and Maintenance
Integrated Process: Teaching/Learning
Content Area: Adult Health/Cardiovascular
Reference: Christensen, B., & Kockrow, E. (2003). Adult health nursing (4th ed.). St. Louis:
Mosby, pp. 320-321.

1286. A client has experienced an episode of pulmonary edema. The nurse determines that the
client’s respiratory status is improving after this episode if which of the following breath sounds
is noted?
1. Crackles throughout the lung fields
2. Crackles in the lung bases
3. Wheezes
4. Rhonchi
PN~Comp~Review-CD-1201-1300 45

Answer: 2
Rationale: Pulmonary edema is characterized by extreme breathlessness, dyspnea, air hunger,
and the production of frothy pink-tinged sputum. Auscultation of the lungs reveals crackles
throughout the lung fields. As the client’s condition improves, the amount of fluid in the alveoli
decreases, and may be detected by crackles in the bases. (Clear lung sounds would indicate full
resolution of the episode). Wheezes and rhonchi are not associated with pulmonary edema.
Test-Taking Strategy: Use the process of elimination. Fluid in the lungs from pulmonary edema
produces sounds that are called crackles, which eliminates options 3 and 4. Eliminate option 1
noting the key words respiratory status is improving. Crackles throughout lung fields do not
indicate an improvement in client condition. Review the clinical manifestations associated with
pulmonary edema if you had difficulty with this question.
Level of Cognitive Ability: Analysis
Client Needs: Physiological Integrity
Integrated Process: Nursing Process/Evaluation
Content Area: Adult Health/Cardiovascular
Reference: Christensen, B., & Kockrow, E. (2003). Adult health nursing (4th ed.). St. Louis:
Mosby, p. 391.

1287. A client with pulmonary edema has an order to receive morphine sulfate intravenously.

The licensed practical nurse assisting in caring for the client determines that the client

experienced an intended effect of the medication if which of the following data are noted?

1. Relief of apprehension
2. Decreased urine output
3. Increased pulse rate
4. Increased blood pressure
Answer: 1
Rationale: Morphine sulfate reduces anxiety and dyspnea in the client with pulmonary edema. It
also promotes peripheral vasodilation and causes blood to pool in the periphery. It decreases
pulmonary capillary pressures, which reduces fluid migration into the alveoli. The client
receiving morphine sulfate is monitored for signs and symptoms of respiratory depression and
extreme drops in blood pressure, especially when the morphine is administered intravenously.
Options 2, 3, and 4 are unrelated to the action of morphine sulfate.
Test-Taking Strategy: Use the process of elimination noting the key words intended effect.
Thinking about the intended effect of treatment for pulmonary edema and the effect of morphine
sulfate will direct you to option 1. Review the effects of this medication if you had difficulty
with this question.
Level of Cognitive Ability: Analysis
Client Needs: Physiological Integrity
Integrated Process: Nursing Process/Evaluation
Content Area: Adult Health/Cardiovascular
Reference: Black, J., & Hawks, J. (2005). Medical-surgical nursing: Clinical management for
positive outcomes (7th ed.). Philadelphia: W.B. Saunders, p. 1880.
PN~Comp~Review-CD-1201-1300 46

1288. A nurse is reinforcing discharge teaching for a client with myocardial infarction (MI) who
will be discharged home instructed to take aspirin. The nurse determines that the client
understands the use of this medication if the client makes which statement?
1. “I will take aspirin until I feel better.”
2. “I will take aspirin every other day.”
3. “I will take aspirin only when I have pain.”
4. “I will take aspirin every day.”
Answer: 4
Rationale: A single daily dose of one baby aspirin (low-dose aspirin) may be a component of the
standard treatment regimen for the client after MI. Aspirin helps to prevent clotting and may
prevent a thrombosis that could cause a second MI. The other three options are unacceptable
because the benefit results from taking the medication on a daily basis.
Test-Taking Strategy: Use the process of elimination. Recalling the action and purpose of
aspirin in the client with MI will direct you to option 4. Review the purpose of aspirin in the
client with a cardiac disorder if you had difficulty with this question.
Level of Cognitive Ability: Analysis
Client Needs: Health Promotion and Maintenance
Integrated Process: Nursing Process/Evaluation
Content Area: Adult Health/Cardiovascular
Reference: Black, J., & Hawks, J. (2005). Medical-surgical nursing: Clinical management for
positive outcomes (7th ed.). Philadelphia: W.B. Saunders, p. 1637.

1289. A client receiving total parenteral nutrition (TPN) is demonstrating signs and symptoms of
an air embolism. What is the first action by the nurse?
1. Notify the physician
2. Stop the TPN

3. Place the client in high-Fowler’s position


4. Place the client on the left side in Trendelenburg position
Answer: 4
Rationale: Lying on the left side may prevent air from flowing into the pulmonary veins.
Trendelenburg position increases intrathoracic pressure, which decreases the amount of blood
pulled into the vena cava during inspiration. The registered nurse is notified and then contacts the
physician, but this is not the first action. Stopping the TPN will not treat the problem.
Test-Taking Strategy: Use the process of elimination and note the key word first in the stem of
the question. This will assist in eliminating option 1. Eliminate option 2 next because this action
will not treat the problem. From the remaining options, think about the principles of gravity and
the anatomy of the cardiopulmonary system to direct you to option 4. Review care to the client
with an air embolism if you had difficulty with this question.
Level of Cognitive Ability: Application
Client Needs: Physiological Integrity
Integrated Process: Nursing Process/Implementation
Content Area: Fundamental Skills
Reference: deWit, S. (2005). Fundamental concepts and skills for nursing. Philadelphia: W.B.
Saunders, p. 703.
PN~Comp~Review-CD-1201-1300 47

1290. A licensed practical nurse (LPN) assisting a registered nurse in the cardiac care unit
(CCU) prepares to admit a client with a diagnosis of myocardial infarction (MI). The LPN
would be certain to have which of the following readily available when the client arrives on the
unit by stretcher?
1. Trapeze bar
2. Bedside commode
3. Electrocardiogram machine
4. Oxygen cannula and flow meter
Answer: 4
Rationale: The client will require oxygen therapy following myocardial infarction to decrease
pain due to ischemia and help minimize the risk of its recurrence. Oxygen decreases pain to the
myocardium by improving oxygen supply. An electrocardiogram machine and bedside
commode may be used but are not priorities at this time. An overhead trapeze could actually be
harmful by causing the client to use the Valsalva maneuver while pulling up in bed.
Test-Taking Strategy: Use the ABCs—airway, breathing, and circulation—to answer the
question. This will direct you to option 4. Review immediate care to the client with an MI if
you had difficulty with this question.
Level of Cognitive Ability: Application
Client Needs: Physiological Integrity
Integrated Process: Nursing Process/Implementation
Content Area: Adult Health/Cardiovascular
Reference: Linton, A., & Maebius, N. (2003). Introduction to medical-surgical nursing (3rd
ed.). Philadelphia: W.B. Saunders, p. 584.

1291. An older client takes cascara sagrada for ongoing management of chronic constipation.
The nurse monitors the client’s laboratory results for which electrolyte imbalance related to long-
term use of this medication?
1. Hypokalemia
2. Hyperkalemia

3. Hyponatremia
4. Hypernatremia
Answer: 1
Rationale: Hypokalemia can result from long-term use of cascara sagrada, a laxative. The
medication stimulates peristalsis and alters fluid and electrolyte transport, thus helping fluid to
accumulate in the colon. Options 2, 3, and 4 are not specifically associated with the use of this
medication.
Test-Taking Strategy: Use the process of elimination. Recalling that this medication is a
laxative will direct you to option 1. Review the side effects of this medication if you had
difficulty with this medication.
Level of Cognitive Ability: Analysis
Client Needs: Physiological Integrity
Integrated Process: Nursing Process/Data Collection
Content Area: Pharmacology
PN~Comp~Review-CD-1201-1300 48

Reference: Hodgson, B., & Kizior, R. (2005). Saunders nursing drug handbook 2005.
Philadelphia: W.B. Saunders, p. 174.

1292. A client has been prescribed cyclobenzaprine (Flexeril) for the management of muscle
spasms in the cervical spine. The client is experiencing drowsiness, dizziness, and dry mouth.
The nurse interprets that these:
1. Represent an allergic reaction to the medication
2. Are the most common side effects of this medication
3. Effects are dose-related; the client should cut the medication dose in half
4. Effects are related to the problem with the cervical spine
Answer: 2
Rationale: Drowsiness, dizziness, and dry mouth are the most common side effects of
cyclobenzaprine. This medication is a centrally-acting skeletal muscle relaxant used in the
management of muscle spasm that accompanies a variety of conditions. Options 1, 3, and 4 are
incorrect.
Test-Taking Strategy: Use the process of elimination. Eliminate option 3 first because the client
is not told to adjust a medication dose. Recalling the signs of an allergic reaction will assist in
eliminating option 1. From the remaining options, focusing on the issue—cyclobenzaprine—
will direct you to option 2. Review the side effects of this medication if you had difficulty with
this question.
Level of Cognitive Ability: Analysis
Client Needs: Physiological Integrity
Integrated Process: Nursing Process/Evaluation
Content Area: Pharmacology
Reference: Hodgson, B., & Kizior, R. (2005). Saunders nursing drug handbook 2005.
Philadelphia: W.B. Saunders, p. 272.

1293. A client with angina pectoris has recently started medication therapy with nitroglycerin.
In planning care for this client, the nurse would place priority on measuring:
1. Therapeutic serum drug levels
2. Serum glucose level
3. Vital signs
4. Intake and output
Answer: 3
Rationale: The nurse would place priority on measuring vital signs, especially the blood pressure
because of the vasodilator action of the medication. Drug levels are not measured for
nitroglycerin, and the medication does not affect serum glucose level. Intake and output may be
measured as part of the general plan of care for the client with heart disease, but it is not directly
related to administration of this medication.
Test-Taking Strategy: Use the process of elimination. Recalling the action of nitroglycerin will
direct you to option 3. Review this medication if you had difficulty with this question.
Level of Cognitive Ability: Application
Client Needs: Physiological Integrity
Integrated Process: Nursing Process/Planning
Content Area: Adult Health/Cardiovascular
PN~Comp~Review-CD-1201-1300 49

Reference: Hodgson, B., & Kizior, R. (2005). Saunders nursing drug handbook 2005.
Philadelphia: W.B. Saunders, p. 779.

1294. Enoxacin (Penetrex) is prescribed for a client with cervical gonorrhea. A nurse is teaching
the client about the medication and tells the client to take the medication:
1. With milk
2. With meals
3. One hour before meals
4. With an antacid
Answer: 3
Rationale: Enoxacin is administered 1 hour before or 2 hours after meals. Antacids may
decrease absorption and should be administered at least 4 hours before or 2 hours after the
medication.
Test-Taking Strategy: Use the process of elimination. Note that options 1, 2, and 4 are similar in
that they indicate administration of the medication with another substance (milk, meals, antacid).
Review this medication if you had difficulty with this question.
Level of Cognitive Ability: Application
Client Needs: Physiological Integrity
Integrated Process: Teaching/Learning
Content Area: Pharmacology
Reference: Skidmore-Roth, L. (2005). Mosby’s drug guide for nurses (6th ed.). St. Louis:
Mosby, p. 308.

1295. A nurse determines that a client with coronary artery disease (CAD) has the necessary
understanding of disease management if the client makes which statement?
1. “It doesn’t matter if my father had high cholesterol.”
2. “My weight has nothing to do with this disease.”
3. “I will walk for one-half hour daily.”
4. “As long as I exercise I can eat anything I wish.”
Answer: 3
Rationale: Lack of physical exercise contributes to the development of CAD, and engaging in a
regular program of exercise helps retard progression of atherosclerosis by lowering cholesterol
levels and developing collateral circulation to heart tissue. Options 2 and 4 are incorrect because
obesity and a diet high in fat can contribute to CAD. Option 1 is incorrect because genetic
factors also contribute to CAD.
Test-Taking Strategy: Use the process of elimination. Recalling the risk factors associated with
CAD and the measures to prevent this disorder will direct you to option 3. Review this content if
you had difficulty with this question.
Level of Cognitive Ability: Comprehension
Client Needs: Health Promotion and Maintenance
Integrated Process: Nursing Process/Evaluation
Content Area: Adult Health/Cardiovascular
Reference: Christensen, B., & Kockrow, E. (2003). Adult health nursing (4th ed.). St. Louis:
Mosby, p. 308.
PN~Comp~Review-CD-1201-1300 50

1296. A nurse has provided instructions to a client receiving enalapril maleate (Vasotec). Which
statement by the client indicates a need for further education?
1. “I need to rise slowly from a lying to sitting position.”
2. “I need to notify the physician if nausea occurs.”
3. “I need to notify the physician if a sore throat occurs.”
4. “I know that several weeks of therapy may be required for the full therapeutic effect.”
Answer: 2
Rationale: If nausea occurs, it is not necessary to notify the physician. The client should be
instructed to consume noncaffeinated carbonated beverages, unsalted crackers, or dry toast to
alleviate the nausea. To reduce the hypotensive effect of this medication, the client is instructed
to rise slowly from a lying to a sitting position and to permit the legs to dangle from the bed
momentarily before standing. The client should report signs of a sore throat or fever to the
physician because these may indicate infection. The client should be notified that several weeks
may be needed for the full therapeutic effect of blood pressure reduction. The client should also
be instructed not to skip doses or discontinue the medication, because severe rebound
hypertension can occur.
Test-Taking Strategy: Note the key words need for further education. These words indicate a
false response question and that you need to select the incorrect client statement. Knowing that
this medication is an antihypertensive will assist in eliminating options 1 and 4. From the
remaining options, note that option 3 indicates the possibility of infection requiring the need for
physician notification. Review this medication if you had difficulty with this question.
Level of Cognitive Ability: Analysis
Client Needs: Health Promotion and Maintenance
Integrated Process: Teaching/Learning
Content Area: Pharmacology
Reference: Hodgson, B., & Kizior, R. (2005). Saunders nursing drug handbook 2005.
Philadelphia: W.B. Saunders, p. 373.

1297. Enoxaparin sodium (Lovenox) is prescribed for the client following hip replacement
surgery. The nurse prepares to have which of the following available in the event that an
overdose of the medication occurs?
1. Vitamin K
2. Protamine sulfate
3. Epinephrine
4. Adrenalin
Answer: 2
Rationale: Enoxaparin sodium is an anticoagulant. Accidental overdose of this medication may
lead to bleeding complications. The antidote is protamine sulfate. Vitamin K is the antidote for
warfarin sodium (Coumadin). Epinephrine and adrenalin are the same medications and are
normally used to treat hypersensitivity reactions, or acute bronchial asthma attacks and
bronchospasms.
Test-Taking Strategy: Use the process of elimination. Eliminate options 3 and 4 first because
they are the same medications. Knowledge that enoxaparin sodium is an anticoagulant and that
the antidote is the same as for heparin will direct you to option 2. Review this medication if you
had difficulty with this question.
Level of Cognitive Ability: Application
PN~Comp~Review-CD-1201-1300 51

Client Needs: Physiological Integrity


Integrated Process: Nursing Process/Planning
Content Area: Pharmacology
Reference: Hodgson, B., & Kizior, R. (2005). Saunders nursing drug handbook 2005.
Philadelphia: W.B. Saunders, p. 377.

1298. A nurse provides home care instructions to a client with sickle cell anemia. Which
statement by the client indicates a need for further instruction?
1. “When I’m feeling better, I’m returning to the soccer team.”
2. “I’m using a schedule to maintain my increased fluid intake.”
3. “I’m going to take a painting class.”
4. “I’ve learned to knit and sew my own clothes.”
Answer: 1
Rationale: Clients with sickle cell anemia are advised to avoid strenuous activities. Quiet
activities as tolerated are recommended when the client is feeling well. Increasing fluid intake is
encouraged to assist in preventing sickle cell crisis.
Test-Taking Strategy: Use the process of elimination noting the key words need for further
instruction. These words indicate a false response question and that you need to select the
incorrect client statement. Eliminate options 3 and 4 first because they are similar. From the
remaining options, recall that fluids are important. Review the client teaching points related to
sickle cell anemia if you had difficulty with this question.
Level of Cognitive Ability: Comprehension
Client Needs: Health Promotion and Maintenance
Integrated Process: Teaching/Learning
Content Area: Child Health
Reference: Wong, D., & Hockenberry, M. (2003). Nursing care of infants and children (7th ed.).
St. Louis: Mosby, pp. 1555-1556.

1299. A nurse is collecting data on a female client who complains of fatigue, weakness, malaise,
muscle pain, joint pain at multiple sites, anorexia, and photosensitivity. Systematic lupus
erythematosus (SLE) is suspected. The nurse further checks for which of the following that is
also indicative of the presence of SLE?
1. Two hemoglobin S genes
2. Ascites
3. Emboli
4. Butterfly rash on cheeks and bridge of nose
Answer: 4
Rationale: SLE is a chronic inflammatory disease that affects multiple body systems. A
butterfly rash on the cheeks and the bridge of nose is a key sign of SLE. Option 1 is found in
sickle cell anemia. Options 2 and 3 are found in many conditions and are not associated with
SLE.
Test-Taking Strategy: Use the process of elimination. Recalling that a key sign of SLE is the
butterfly rash will direct you to option 4. If you are unfamiliar with this disease and the clinical
manifestations, review this disorder.
Level of Cognitive Ability: Analysis
Client Needs: Physiological Integrity
PN~Comp~Review-CD-1201-1300 52

Integrated Process: Nursing Process/Data Collection


Content Area: Adult Health/Immune
Reference: Christensen, B., & Kockrow, E. (2003). Adult health nursing (4th ed.). St. Louis:
Mosby, p. 79.

1300. A nurse is caring for a client with terminal cancer who is close to death. In reviewing the
plan of care, the nurse determines that a priority is to:
1. Keep the client well sedated so the client is totally unaware of what is actually happening
2. Make sure that the family has privacy and is kept informed of what is happening at all times
3. Carry out the physician’s orders so that all prescribed treatments are carried out on time
4. Maintain the client’s dignity and self-esteem and make the client as comfortable as possible
Answer: 4
Rationale: The nurse needs to focus on the needs of the client, keep the client comfortable, and
maintain dignity and self-esteem for the client. While the nurse needs to control the pain, it is
not necessary to keep the client sedated so that the client is totally unaware of what is happening.
The client should be able to interact with family members and make care decisions. Family needs
are important, but the client’s needs are most important. Prescribed treatment needs to be carried
out, but making the client comfortable and maintaining dignity is the priority.
Test-Taking Strategy: Use the process of elimination. Focus on the key word priority. Also note
the client of the question. This will assist in eliminating option 2. Eliminate option 1 because of
the words “sedated” and “totally unaware.” Eliminate option 3 because of the word “all.”
Options 1, 2, and 3 may be incorporated into the care but option 4 provides a client- focused
care: that is the priority. Review care to the client with a terminal illness if you had difficulty
with this question.
Level of Cognitive Ability: Application
Client Needs: Psychosocial Integrity
Integrated Process: Caring
Content Area: Fundamental Skills
Reference: Christensen, B., & Kockrow, E. (2003). Adult health nursing (4th ed.). St. Louis:
Mosby, p. 737.

You might also like